You are on page 1of 95

General Studies

Paper#1-
Sectional
Paper#1

Target PCS Lucknow

https://targetpcslucknow.com/

Whatsapp/Call @ 7390023092
General Studies Paper#1-Sectional Paper#1

Time Allowed: 2 HoursMaximum Marks: 200

INSTRUCTIONS

1. IMMEDITELY AFTER THE COMMENCEMENT OF THE EXAMINATION, YOU SHOULD

CHECK THAT THIS TEST BOOKLET DOES NOT HAVE ANY UNPRINTED OR TORN OR MISSING PAGES OR
ITEMS, ETC. IF SO, GET IT REPLACED BY A COMPLETE TEST BOOKLET.

2. You have to enter your Roll Number on the Test

Booklet in the Box provided alongside. DO NOT

Write anything else on the Test Booklet.

4. This Test Booklet contains 150 items (questions). Each item is printed only in English. Each item
comprises four responses (answers). You will select the response which you want to mark on the
Answer Sheet. In case you feel that there is more than one correct response, mark the response
which you consider the best. In any case, choose ONLY ONE response for each item.

5. You have to mark all your responses ONLY on the separate Answer Sheet provided. See directions
in the Answer Sheet.

6. All items carry equal marks.

7. Before you proceed to mark in the Answer Sheet the response to various items in the Test
Booklet, you have to fill in some particulars in the Answer Sheet as per instructions sent to you with
your Admission Certificate.

8. After you have completed filling in all your responses on the Answer Sheet and the examination
has concluded, you should hand over to the Invigilator only the Answer Sheet. You are permitted to
take away with you the Test Booklet.

9. Sheets for rough work are appended in the Test Booklet at the end.

10. Penalty for wrong answers:

THERE WILL BE PENALTY FOR WRONG ANSWERS MARKED BY A CANDIDATE IN THE OBJECTIVE TYPE
QUESTION PAPERS.

(i) There are four alternatives for the answer to every question. For each question for which a wrong
answer has been given by the candidate, one-third of the marks assigned to that question will be
deducted as penalty.

(ii) If a candidate gives more than one answer, it will be treated as a wrong answer even if one of the
given answers happens to be correct and there will be same penalty as above to that question.

(iii) If a question is left blank, i.e., no answer is given by the candidate, there will be no penalty for
that question.

Target PCS Lucknow Page 1


General Studies Paper#1-Sectional Paper#1

पूर्व ऐतिहासिक काल

1. भारतीय प्रागिततहास के जनक के रूप में ककसे जाना जाता है ?

A. ले मेसरु रयर

B. रॉबर्ट ब्रस
ू फूर्े

C. सब्ु बा राव

D. माइल्स बक
ु ीर्ा

उत्तर. B

स्पष्र्ीकरण: "भारतीय प्रागिततहास का पपता" पद उस व्यक्तत को ददया िया था जो भारतीय नहीीं है ,


ब्रब्रदर्श भपू वज्ञानी रॉबर्ट ब्रस
ू फूर्े के अततररतत प्रयासों और कडी मेहनत ने उन्हें भारतीय प्रागिततहास
का पपता बना ददया। फूर्े ने भारत में कई प्रािैततहाससक परु ापाषाणकालीन कलाकृततयों की खोज की।

2. दक्षिण भारत के महापाषाण मख्


ु य रूप से सींबगीं ित हैं-

A. मध्यपाषाण काल के साथ

B. लौह यि
ु के साथ

C. ताम्रपाषाण यि
ु के साथ

D. नवपाषाण यि
ु के साथ

उत्तर. B

स्पष्र्ीकरण: महापाषाण भारतीय उपमहाद्वीप में फैले हुए हैं, हालाींकक उनमें से अगिकाींश प्रायद्वीपीय
भारत में पाए जाते हैं, जो महाराष्र (मख्
ु य रूप से पवदभट), कनाटर्क, तसमलनाडु, केरल, आींध्र प्रदे श
और तेलि
ीं ाना राज्यों में केंदित हैं। जब उत्तर भारत ताम्रपाषाण काल से िज
ु रा, तो यह दक्षिण भारत
में लौह यि
ु की सींस्कृतत का चरण था। लौह यि
ु सींिम यि
ु से पहले का था; हालााँकक लौह यि
ु की
सींस्कृतत का बाद का दहस्सा सींिम यि
ु के साथ आिे बढा और इसे दतकन (Deccan) और दक्षिण
भारत में महापाषाण सींस्कृतत के रूप में जाना जाता है ।

Target PCS Lucknow Page 2


General Studies Paper#1-Sectional Paper#1

3. भारत में परु ापाषाण काल को उपकरण प्रौद्योगिकी के आिार पर तीन चरणों में बाींर्ा िया है ।
तनम्नसलखखत में से कौन-सा/से चरण इसके औजारों से सही सम
ु ेसलत है /हैं?

A. मध्य परु ापाषाण – िच्


ु छे (flakes) पर बना उपकरण

B. ऊपरी परु ापाषाण काल - फ्लेतस और ब्लेड पर बने उपकरण

C. तनचला परु ापाषाण- हस्त कुल्हाडी (handaxe) और तलीवर (cleaver) उद्योि

D. उपरोतत सभी

उत्तर. D

स्पष्र्ीकरण: तनचले परु ापाषाण काल के सींस्कृतत चरण में मख्


ु य उपकरण प्रकार हाथ की कुल्हाडी और
तलीवर थे, साथ ही चॉपर-चॉपपींि उपकरण भी थे। उन्हें कोर(cores) और फ्लेतस(flakes) दोनों पर
बनाया िया था।

4. तनम्नसलखखत में से कौन सा कथन सही नहीीं है ?

A. भारत में परु ापाषाण काल का मनष्ु य आि का उपयोि जानता है ।

B. उत्तर भारत में ताम्र यि


ु और प्रारीं सभक लौह यि
ु की पहचान की जा सकती है । हालाींकक, लौह यि

दक्षिण भारत में पाषाण युि के तरु ीं त बाद आता है ।

C. कैमरू पवटतमाला और समजाटपरु क्जले में प्रािैततहाससक काल के गचत्र समले हैं।

D. दक्षिण भारत के िर्


ुीं ू र और कुरनल
ू क्जलों में परु ापाषाण काल के अनाज पाए िए हैं।

उत्तर. A

स्पष्र्ीकरण: मध्य परु ापाषाण यि


ु मख्
ु य रूप से मनष्ु य के प्रारीं सभक रूप तनएींडरथल (Neanderthal)
से जुडा था, क्जसके अवशेष अतसर िफ
ु ाओीं में आि के उपयोि के प्रमाण के साथ पाए जाते हैं। उन्हें
अपना नाम तनएींडर (जमटनी) की घार्ी से समला। तनएींडरथल प्रािैततहाससक काल के सशकारी थे। भारत
में , परु ापाषाण काल के परु
ु ष आि के उपयोि को नहीीं जानते थे।

5. तनम्नसलखखत में से कौन-सी सींस्कृतत समट्र्ी के बतटनों को रीं िना शरू


ु करने वाली पहली सींस्कृतत
थी?

A. मध्य पाषाण

Target PCS Lucknow Page 3


General Studies Paper#1-Sectional Paper#1

B. लौह यि

C. नव पाषाण

D. ताम्रपाषाण

उत्तर. D

स्पष्र्ीकरण: गचब्रत्रत समट्र्ी के बतटनों की उपक्स्थतत ताम्रपाषाण काल की पहचान है । समट्र्ी के बतटनों
में लाल बतटन, िहरे लाल रीं ि के बतटन से लेकर िहरे भरू े और काले, गचत्रात्मक लाल और काले और
पॉसलश ककए हुए लाल रीं ि के बतटन होते हैं।

6. सच
ू ी I को सच
ू ी II से सम
ु ेसलत कीक्जए और नीचे ददए िए कूर् से सही उत्तर का चयन कीक्जए:

सच
ू ी I सच
ू ी II

(आय)ु (घर्ना)

a. परु ापाषाण यि
ु 1. ताींबा जाना जाता था

b. मध्यपाषाण यि
ु 2. आितु नक मनष्ु य का उदय

c. नवपाषाण यि
ु 3. आि की खोज

d. ताम्रपाषाण यि
ु 4. फसलों की कर्ाई के सलए औजारों का उपयोि ककया जाता था

कोड:

a b c d

A. 4 3 2 1

B. 3 2 4 1

C. 2 3 4 1

Target PCS Lucknow Page 4


General Studies Paper#1-Sectional Paper#1

D. 3 4 1 2

उत्तर. C

7. असभकथन (A): नवपाषाण यि


ु के दौरान, दफन अवशेषों के साथ समले मोततयों से पता चलता है
कक लोि लैपपस लाजुली, कारे सलयन, बींिे हुए सल
ु ेमानी (banded agate) और सफेद समि
ु ी खोल से
बने मोततयों का इस्तेमाल करते थे।

कारण (R): एक ताींबे का मनका भी समला है । मोती से बने खोल की चूडडयों और लर्कन (pendant)
का होना लींबी दरू ी के व्यापार को इींगित करता है ।

उपरोतत के सन्दभट में तनम्नसलखखत में से कौन सा सही है

A. A और R दोनों सत्य हैं और R, A की सही व्याख्या करता है ।

B. A और R दोनों सत्य हैं लेककन R, A की सही व्याख्या नहीीं है ।

C. A सच है लेककन R िलत है ।

D. A िलत है लेककन R सत्य है

उत्तर. B

स्पष्र्ीकरण: नवपाषाण यि
ु , क्जसका अथट है नया पाषाण यि
ु , पाषाण यि
ु का अींततम और तीसरा
भाि था। भारत में , यह लिभि 7,000 ईसा पव
ू ट से 1,000 ईसा पव
ू ट तक फैला था। नवपाषाण यि

की मख्
ु य पवशेषता बसे हुए कृपष का पवकास और पॉसलश ककए िए पत्थरों से बने औजारों और
हगथयारों का उपयोि है । इस अवगि के दौरान उिाई जाने वाली प्रमख
ु फसलें रािी, चने की दाल,
कपास, चावल, िेहूीं और जौ थीीं। इस यि
ु में पहली बार समट्र्ी के बतटन ददखाई ददए।

8. प्रािैततहाससक शैल कला (rock art) के तनम्नसलखखत में से कौन से महत्वपण


ू ट शैल गचत्र (rock
painting) स्थल हैं?

1. उत्तर प्रदे श में मरु हाना पहाड

2. मध्य प्रदे श में भीमबेर्का, आदमिढ, लखहा जुआर

3. कनाटर्क में कुपािल्लू

Target PCS Lucknow Page 5


General Studies Paper#1-Sectional Paper#1

4. उत्तर-पक्चचम पाककस्तान में चारिल


ु ।

नीचे ददए िए कूर्ों से उत्तर चुतनए:

A. 1, 2, 3 और 4

B. 1, 3 और 4

C. 1, 2 और 3

D. 2, 3 और 4

उत्तर. A

स्पष्र्ीकरण: भारत में लिभि सभी शैल-आश्रय (rock shelter) ऊपरी परु ापाषाण और मध्य पाषाण
काल के लोिों द्वारा बसे हुए थे। उनमें शैल गचत्र शासमल हैं जो पवसभन्न पवषयों, मख्
ु य रूप से
जानवरों, या लोिों और जानवरों दोनों सदहत दृचयों को दशाटती हैं। इन शैलगचत्रों का पवतरण बहुत
पवस्तत
ृ है । वे उत्तर-पक्चचम पाककस्तान में चारिल
ु से पव
ू ट में उडीसा तक और उत्तर में कुमाऊीं की
पहाडडयों से लेकर दक्षिण में केरल तक पाए िए हैं।

9. तनम्नसलखखत में से कौन सा/से कथन िलत है /हैं?

1. नवपाषाण यि
ु के लोि पहले खाद्य उत्पादक थे।

2. नवपाषाण यि
ु के मनष्ु यों को कुम्हार का पदहया ज्ञात नहीीं था।

3. नवपाषाण यि
ु के लोि पॉसलश ककए हुए पत्थर के औजारों का इस्तेमाल करते थे।

नीचे ददए िए कूर् का प्रयोि कर सही उत्तर चुतनए:

A. 2 केवल

B. केवल 1 और 2

C. केवल 3 3

D. 1, 2 और 3

उत्तर. A

Target PCS Lucknow Page 6


General Studies Paper#1-Sectional Paper#1

स्पष्र्ीकरण: नवपाषाण काल के लोि सबसे प्राचीन कृषक समद


ु ाय थे। सबसे पहले नवपाषाण काल के
तनवाससयों ने घरे लू पशओ
ु ीं को पाला और अनाज का उत्पादन ककया। नवपाषाण यि
ु के मनष्ु य समट्र्ी
के बतटन बनाने के पदहया के बारे में जानते हैं। नवपाषाण यि
ु के लोि पॉसलश ककए हुए पत्थर के
औजारों और उपकरणों का इस्तेमाल करते थे। वे पवशेष रूप से पत्थर की कुल्हाडडयों का इस्तेमाल
करते थे, जो भारत के पहाडी इलाकों के एक बडे दहस्से में बडी सींख्या में पाए िए हैं।

10. ताम्रपाषाण काल के दौरान भारत में जीवन के बारे में तनम्नसलखखत कथनों पर पवचार करें :

1. ताम्रपाषाण काल का मनष्ु य सलपप से पररगचत था।

2. मदहलाएीं खोल और हड्डी के आभष


ू ण पहनती थीीं और अपने बालों में बारीक काम की हुई कींघी
रखती थीीं।

3. मनष्ु य ने कपडे की कताई और बन


ु ाई की तकनीक का आपवष्कार ककया था।

4. मौज-मस्ती और मनोरीं जन के सलए अवकाश के घींर्े ददए िए थे।

ऊपर ददए िए कथनों में से कौन-सा/से सही है /हैं?

A. केवल 1 और 2

B. केवल 1, 2 और 3

C. केवल 2, 3 और 4

D. उपरोतत सभी

उत्तर. D

स्पष्र्ीकरण: ताम्रपाषाण काल या नवपाषाण काल िातओ


ु ीं के उपयोि का प्रतीक है , क्जसमें तााँबा
सबसे पहले इस्तेमाल होने वाली िातु थी। ताम्रपाषाण का अथट है ताींबे और पत्थर का उपयोि। तााँबा
शायद मानव द्वारा इस्तेमाल की जाने वाली पहली िातु थी और ताम्र यि
ु की अवगि 1800-800
ईसा पव
ू ट थी। इस प्रकार, ताम्रपाषाण काल पाषाण यि
ु और िातु यि
ु के बीच का पररवतटनशील काल
था। यह वह दौर था जब पत्थर के औजार अपना महत्व खो रहे थे और ताींबे के औजारों को आिार
समल रहा था। ताम्रपाषाण काल के लोि आभष
ू ण और सजावर् के शौकीन थे। मदहलाएीं खोल और
हड्डी के आभष
ू ण पहनती थीीं और अपने बालों में बारीक काम की हुई कींघी रखती थीीं। उन्होंने

Target PCS Lucknow Page 7


General Studies Paper#1-Sectional Paper#1

कारे सलयन (carnelian), शैलखर्ी (steatite) और तवाट्टज किस्र्ल (quartz crystal) जैसे अिट-
कीमती पत्थरों के मोततयों का तनमाटण ककया था।

सििंधु घाटी िभ्यिा

11. कृब्रत्रम ईंर् डॉकयाडट (artificial brick dockyard) वाला एकमात्र ससींिु स्थल कौन सा था?

A. लोथल

B. कालीबींिा

C. हडप्पा

D. मोहनजोदडो

उत्तर. A

स्पष्र्ीकरण: लोथल ससींिु घार्ी सभ्यता का बींदरिाह शहर था। यह िज


ु रात के सरिवाला में क्स्थत
था। िाद के जमाव से बचने के सलए िोदी बाडा (dockyard) मख्
ु य िारा से दरू क्स्थत था। यह
अनम
ु ान लिाया जाता है कक लोथल इींजीतनयरों ने ज्वार की िततपवगियों और ईंर्-तनसमटत सींरचनाओीं
पर उनके प्रभावों का अध्ययन ककया, तयोंकक समली दीवारें भट्ठे से जली हुई ईंर्ों की थीीं।

12. ब्रबना ककलेबद


ीं ी के ससींिु सभ्यता का एकमात्र शहर कौन सा था?

A. कालीबींिा

B. मोहनजोदडो

C. हडप्पा

D. चन्हुदडो

उत्तर. D

स्पष्र्ीकरण: चन्हुदडो एकमात्र हडप्पा शहर है क्जसमें िढवाले िढ (Fortified Citadel) नहीीं हैं।
चन्हुदडो ने पवसभन्न मतू तटयों, मह
ु रों, खखलौनों, हड्डी के औजारों के कारखानों का प्रमाण ददया है ,

Target PCS Lucknow Page 8


General Studies Paper#1-Sectional Paper#1

इससलए यह व्याख्या की िई है कक यह बहुत सारे कारीिरों वाला शहर था और एक औद्योगिक शहर


भी था।

13. ससींिु घार्ी सभ्यता का महान स्नानािार कहााँ पाया जाता है

A. हडप्पा

B. मोहनजो-दारो

C. रोपड

D. कालीबींिा

उत्तर. B

स्पष्र्ीकरण: महान स्नानािार पाककस्तान के ससींि में मोहनजो-दारो में प्राचीन ससींिु घार्ी सभ्यता के
खींडहरों में पाई जाने वाली सबसे प्रससद्ि सींरचनाओीं में से एक है । परु ाताक्त्वक साक्ष्य इींगित करते हैं
कक स्नानािार तीसरी सहस्राब्दी ईसा पव
ू ट में बनाया िया था, "िढ (citadel)" र्ीले के तनमाटण के
तरु ीं त बाद, क्जस पर यह क्स्थत है ।

14. हडप्पा के लोि सबसे शरु


ु आती लोि थे क्जन्होंने पहली बार तनम्नसलखखत में से ककसका उत्पादन
ककया था

A. सील्स (seals)

B. काींस्य उपकरण

C. कपास

D. जौ

उत्तर. C

स्पष्र्ीकरण: ससींिु घार्ी में ककसान कपास की कताई और बन


ु ाई करने वाले पहले व्यक्तत थे। 1929
में परु ातत्वपवदों ने 3250 और 2750 ईसा पव
ू ट के बीच के मोहनजो-दारो, जो अब पाककस्तान है , में
सत
ू ी वस्त्रों के र्ुकडे बरामद ककए। तनकर्वती मेहरिढ में पाए जाने वाले कपास के बीज 5000 ईसा
पव
ू ट के हैं। सादहक्त्यक सींदभट आिे उपमहाद्वीप के कपास उद्योि की प्राचीन प्रकृतत की ओर इशारा

Target PCS Lucknow Page 9


General Studies Paper#1-Sectional Paper#1

करते हैं। 1500 और 1200 ईसा पव


ू ट के बीच रगचत वैददक ग्रींथ कपास की कताई और बन
ु ाई का
सींकेत दे ते हैं।

15. तनम्नसलखखत में से कौन सा हडप्पा स्थल िज


ु रात में क्स्थत नहीीं है ?

A. सरु कोर्दा

B. रीं िपरु

C. सत्ु कािें डोर

D. दे सलपरु

उत्तर. C

स्पष्र्ीकरण: सत
ु कािनडोर (या सत
ु कािेन दोर) ससींिु घार्ी सभ्यता का सबसे पक्चचमी ज्ञात
परु ाताक्त्वक स्थल है । यह कराची से लिभि 480 ककमी पक्चचम में मकरान तर् पर ग्वादर के पास,
ईरानी सीमा के करीब, पाककस्तान के बलगू चस्तान प्राींत में क्स्थत है । यह स्थल दचत नदी के पक्चचमी
तर् के पास है और एक छोर्ी िारा के साथ इसका सींिम है , क्जसे िाजो कौर के नाम से जाना जाता
है । यह पयाटप्त पत्थर की दीवारों और प्रवेश द्वारों के साथ एक छोर्ी बस्ती थी।

16. सच
ू ी I को सच
ू ी II से सम
ु ेसलत कीक्जए और नीचे ददए िए कूर् से सही उत्तर का चयन कीक्जए:

सच
ू ी I सच
ू ी II

(हडप्पा स्थल) (राज्य)

a. मोहनजोदडो 1. हररयाणा

b. लोथल 2. िज
ु रात

c. कालीबींिा 3. ससींि

d. बनवाली 4. राजस्थान

कोड:

a b c d

Target PCS Lucknow Page 10


General Studies Paper#1-Sectional Paper#1

A. 4 3 2 1

B. 3 2 4 1

C. 2 3 4 1

D. 3 4 1 2

उत्तर. B

17. असभकथन (A): ससींिु सभ्यता में मींददरों या सावटजतनक पज


ू ा स्थलों की उपक्स्थतत का कोई
तनणाटयक सबत
ू नहीीं है ।

कारण (R): हडप्पावाससयों के सलए िमट शायद सावटजतनक मामले से अगिक व्यक्ततित और तनजी
मामला था।

उपरोतत के सन्दभट में तनम्नसलखखत में से कौन सा सही है

A. A और R दोनों सत्य हैं और R, A की सही व्याख्या करता है ।

B. A और R दोनों सत्य हैं लेककन R, A की सही व्याख्या नहीीं है ।

C. A सच है लेककन R िलत है ।

D. A िलत है लेककन R सत्य है

उत्तर. C

स्पष्र्ीकरण: ससींिु घार्ी सभ्यता में 1,000 से अगिक शहर और बक्स्तयाीं शासमल थीीं. इन शहरों में
सव्ु यवक्स्थत अपसशष्र् जल तनकासी व्यवस्था, कचरा सींग्रह प्रणाली, और सींभवतः यहाीं तक कक
सावटजतनक अन्न भींडार और स्नानािार भी शासमल थे। यद्यपप बडी-बडी दीवारें और िढ(citadel) थे,
कफर भी स्मारकों, महलों या मींददरों का कोई प्रमाण नहीीं है । हडप्पा की कलाकृततयों की एकरूपता से
पता चलता है कक मह
ु रों, बार्ों और ईंर्ों को पवतनयसमत करने के सलए ककसी प्रकार का अगिकार और
शासन था।

18. असभकथन (A): ससींिु घार्ी सभ्यता के दौरान पीपल के पेड की पज


ू ा की जाती थी।

Target PCS Lucknow Page 11


General Studies Paper#1-Sectional Paper#1

कारण (R): ससींिु घार्ी सभ्यता के दौरान प्रोर्ो-सशव (proto-shiv) पींथ लोकपप्रय था।

उपरोतत के सन्दभट में तनम्नसलखखत में से कौन सा सही है

A. A और R दोनों सत्य हैं और R, A की सही व्याख्या करता है ।

B. A और R दोनों सत्य हैं लेककन R, A की सही व्याख्या नहीीं है ।

C. A सच है लेककन R िलत है ।

D. A िलत है लेककन R सत्य है

उत्तर. B

स्पष्र्ीकरण: ससींिु घार्ी सभ्यता के स्थलों से खोजी िई पशप


ु तत मह
ु र की पहचान प्रोर्ो-सशव मह
ु र के
रूप में की िई है । हडप्पावाससयों ने भी पीपल के पेडों की पज
ू ा की होिी। एक मह
ु र में पेड को प्रणाम
करते हुए सात आकृततयों को दशाटया िया है । पेड पर एक सीींि वाली आकृतत खडी है । कुछ पवद्वानों
का तकट है कक यह दृचय बाद के सप्तमाब्रत्रकों की याद ददलाता है ।

19. हडप्पा के शहरों के बारे में तनम्नसलखखत कथनों को दे खें:

1. सडकें सीिी चलती थीीं और एक दस


ू रे को समकोण पर कार्ती थीीं।

2. मकान ईंर्ों के बने होते थे और उनकी मोर्ी-मोर्ी दीवारें होती थीीं, क्जन पर प्लास्र्र और रीं ि चढा
होता था।

3. घरों की छतें शींतवाकार थीीं।

4. घरों में बहुत सारी खखडककयााँ थीीं।

ऊपर ददए िए कथनों में से कौन-सा/से सही है /हैं?

A. केवल 1 और 2

B. केवल 1, 2 और 3

C. केवल 1, 2 और 4

D. उपरोतत सभी

Target PCS Lucknow Page 12


General Studies Paper#1-Sectional Paper#1

उत्तर. A

स्पष्र्ीकरण: हडप्पा सींस्कृतत अपनी निर तनयोजन प्रणाली के कारण दस


ू रों से अलि थी। हडप्पा और
मोहनजोदडो प्रत्येक का अपना िढ (citadel) या एिोपोसलस (acropolis) था, जहााँ सींभवतः शासक
विट के सदस्य रहते थे। प्रत्येक शहर में िढ के नीचे एक तनचला शहर था क्जसमें ईंर् के घर थे,
क्जनमें आम लोि रहते थे। शहरों में घरों की व्यवस्था के बारे में उल्लेखनीय बात यह है कक उन्होंने
गग्रड प्रणाली का पालन ककया था। हडप्पा के निरों में अन्न भींडार एक महत्वपण
ू ट भाि थे। हडप्पा के
शहरों में पकी हुई ईंर्ों का उपयोि उल्लेखनीय है , तयोंकक समस्र की समकालीन इमारतों में मख्
ु य रूप
से सख
ू ी ईंर्ों का उपयोि ककया जाता था। मोहनजोदडो की जल तनकासी व्यवस्था बहुत प्रभावशाली
थी। लिभि सभी शहरों में हर बडे या छोर्े घर का अपना आींिन और स्नानघर होता था। कालीबींिा
में कई घरों में कुएीं थे। िोलावीरा और लोथल (िज
ु रात) जैसे स्थलों पर, परू ी बस्ती को ककलेबद
ीं कर
ददया िया था, और शहर के भीतर के दहस्सों को भी दीवारों से अलि कर ददया िया था।

20. तनम्नसलखखत कथनों पर पवचार करें :

1. बज
ु ह
ट ोम में , घरे लू कुत्तों को उनके मासलकों के साथ दफनाया जाता था।

2. कोल्डीहवा चावल की खेती के सलए जाना जाता है ।

3. पपकलीहाल में राख के र्ीले और आवास स्थल समले हैं।

ऊपर ददए िए कथनों में से कौन-सा/से सही है /हैं?

A. केवल 1

B. केवल 1 और 2

C. केवल 3

D. 1, 2 और 3

उत्तर. D

स्पष्र्ीकरण: बज
ु ह
ट ोम के लोि मोर्े िस
ू र समट्र्ी के बतटनों का प्रयोि करते थे। यह ददलचस्प है कक
बज
ु ह
ट ोम में , घरे लू कुत्तों को उनके मासलकों के साथ उनकी कब्रों में दफनाया िया था। यह प्रथा भारत
में ककसी अन्य नवपाषाण सींस्कृतत में स्पष्र् नहीीं प्रतीत होती है । इलाहाबाद क्जले में कोक्ल्डहवा और
महािरा जैसे नवपाषाण स्थलों को 5वीीं सहस्राब्दी ईसा पव
ू ट में चावल की खेती के सलए जाना जाता है ।

Target PCS Lucknow Page 13


General Studies Paper#1-Sectional Paper#1

पपकलीहाल में नवपाषाण काल के तनवासी पशप


ु ालक थे। उनके पास पालतू पश,ु भेड, बकररयाीं आदद
थे, और वे मौसमी सशपवर लिाते थे, जो खम्भों और डींडों से बने काउपन से तघरे हुए थे, क्जसमें वे
िोबर जमा करते थे। जब उन्हें वहाीं से जाना होता था, तो वे परू े सशपवर-स्थान) कैं पपींि ग्राउीं ड) को
आि लिा दे ते थे और अिले कैं पपींि सत्र के सलए उसे साफ कर दे ते थे। पपकलीहाल में राख के र्ीले
और तनवास स्थान दोनों समले हैं।

र्ैदिक ििंस्कृति

21. ऋग्वेद के भजन कहााँ सलखे िए थे?

A. पींजाब

B. िज
ु रात

C. राजस्थान

D. उत्तर प्रदे श

उत्तर. A

स्पष्र्ीकरण: भजन ऋपषयों (ऋपष) द्वारा रगचत थे, जो उच्च पवद्वान परु
ु ष थे। कुछ भजनों की रचना
मदहलाओीं ने भी की थी। ऋग्वेद सलखने के सलए सींस्कृत के एक परु ाने रूप का इस्तेमाल ककया िया
था। आज हम क्जस सींस्कृत का उपयोि करते हैं, वह ऋग्वेद की सींस्कृत से ब्रबल्कुल अलि है । आयट
खानाबदोश दे हाती लोि थे जो मल
ू रूप से मध्य एसशया के कैक्स्पयन सािर िेत्र में रहते थे। तीसरी
सहस्राब्दी ईसा पव
ू ट के मध्य में , उन्होंने यरू ोप और एसशया की ओर प्रवास करना शरू
ु कर ददया।
चरािाहों की तलाश में इस प्रवासी आबादी का एक विट 2000 ईसा पव
ू ट के आसपास भारतीय
उपमहाद्वीप की सीमाओीं पर पहुींच िया। कफर वे पींजाब में बस िए।

22. 'आयव
ु ेद' की उत्पपत्त में हुई है

A. ऋग्वेद

B. सामवेद

C. यजुवेद

Target PCS Lucknow Page 14


General Studies Paper#1-Sectional Paper#1

D. अथवटवेद

उत्तर. D

स्पष्र्ीकरण: क्जन मल
ू भत
ू ससद्िाींतों पर आयव
ु ेददक प्रणाली आिाररत है , वे अतनवायट रूप से सभी
समय के सलए सत्य हैं और बदलते नहीीं हैं। ये मानवीय कारकों पर, आींतररक कारणों पर आिाररत
हैं। आयव
ु ेद की उत्पपत्त का श्रेय अथवटवद
े को ददया जाता है जहाीं कई बीमाररयों के साथ-साथ उनके
उपचार का उल्लेख ककया िया है ।

23. वेदों के सत
ू तों से सींबगीं ित कमटकाींडों को ____ के रूप में जाना जाता है

A. सींदहता

B. अरण्यकसी

C. ब्राह्मणसी

D. उपतनषद

उत्तर. D

स्पष्र्ीकरण: ब्राह्मण मख्


ु य रूप से िासमटक दस्तावेज हैं, क्जनमें कमटकाींड के उपदे श और बसलदान
कतटव्य शासमल हैं।

24. तनम्नसलखखत में से ककस सशल्प कौशल का अभ्यास आयों द्वारा नहीीं ककया िया था?

A. समट्र्ी के बतटन

B. आभष
ू ण

C. बढईिीरी

D. लोहार

उत्तर. D

Target PCS Lucknow Page 15


General Studies Paper#1-Sectional Paper#1

स्पष्र्ीकरण: लोहार सशल्प कौशल का अभ्यास आयों द्वारा नहीीं ककया जाता था। वैददक यि
ु के आयों
को लौह िातु की जानकारी नहीीं थी। उत्तर वैददक काल के लोि लोहे का प्रयोि करते थे। लोहे के
उपयोि की शरु
ु आत पारीं पररक रूप से उत्तर वैददक लोिों के पव
ू ट की ओर प्रवास से जुडी हुई है ।

25. वैददक काल में राजा लोिों से जो कर वसल


ू करते थे उसे कहा जाता था;

A. बसल

B. पवदथ

C. वमटन

D. कारा

उत्तर. A

स्पष्र्ीकरण: वैददक यि
ु में राजा लोिों से बसल एकत्र करता था जो राजा या भिवान को दी जाने
वाली भें र् होती है । ऋग्वैददक काल में स्वेच्छा से इसका भि
ु तान ककया जाता था लेककन बाद में इसे
अतनवायट कर ददया िया।

26. सच
ू ी I को सच
ू ी II से सम
ु ेसलत करें और नीचे ददए िए कूर् से सही उत्तर का चयन करें :

सच
ू ी I सच
ू ी II

(पस्
ु तक) (पवषय)

a. िमटसत्र
ू 1. िह
ृ स्थों के कतटव्य

b. श्रौतसत्र
ू 2. भारत की सबसे परु ानी कानन
ू की ककताबें

c. िह्
ृ यसत्र
ू 3. यज्ञ के दौरान मींत्र जाप

d. सल
ु वसत्र
ू 4. ज्यासमतत और िखणत की शरु
ु आत का प्रतीक है

कोड:

a b c d

Target PCS Lucknow Page 16


General Studies Paper#1-Sectional Paper#1

A. 4 3 2 1

B. 3 2 4 1

C. 2 3 1 4

D. 3 4 1 2

उत्तर. C

स्पष्र्ीकरण: एक िमट सत्र


ू एक ककताब या शास्त्र है जो दहींद ू कानन
ू की प्रारीं सभक नीींव बनाता है ,
क्जसमें सरकार, जातत, लोिों के बीच सींबि
ीं ों, आगथटक कायों, आहार और िासमटक मामलों के सींबि
ीं में
तनयम शासमल हैं। ऐसे कई ग्रींथ थे क्जन्हें िमट सत्र
ू माना जाता था, लेककन आितु नक समय में केवल
चार ही बचे थे। िौतम िमटसत्र
ू एक सींस्कृत पाठ है और सींभवत: सबसे परु ाने दहींद ू िमटसत्र
ू ों (600-
200 ईसा पव
ू )ट में से एक है , क्जनकी पाींडुसलपपयाीं आितु नक यि
ु में बची हुई हैं। श्रौतसत्र
ू श्रतु त पर
आिाररत कमटकाींड-सींबि
ीं ी सत्र
ू हैं। कल्प (वेदाींि) सत्र
ू ों के पहले सींस्करण सींभवतः छठी शताब्दी ईसा
पव
ू ट द्वारा रगचत थे, लिभि उसी समय से शरू
ु हुए जब वेदों की ब्राह्मण परत की रचना की िई थी
और अगिकाींश अनष्ु ठान सत्र
ू लिभि 300 ईसा पव
ू ट तक परू े हो िए थे। िह्
ृ य-सत्र
ू उन समारोहों
(सींस्कारों) का वणटन करते हैं जो ककसी व्यक्तत के जीवन के प्रत्येक चरण को िभाटिान के िण से
लेकर अींततम मत्ृ यु सींस्कार तक गचक्ह्नत करते हैं, पाींच दै तनक बसलदान (महायज्ञ); मौसमी समारोह;
और क्जन्हें पवशेष अवसरों पर मनाया जाता है , जैसे कक िह
ृ तनमाटण या पशु प्रजनन। चार प्रमख

सल्
ु वा सत्र
ू , जो िखणतीय रूप से सबसे महत्वपण
ू ट हैं, वे बौिायन, मानव, आपस्तींब और कात्यायन
द्वारा रगचत हैं। उनमें से सबसे परु ाना बौिायन का है और 600 ईसा पव
ू ट का है । उन्होंने पाइथािोरस
प्रमेय और पाइथािोरस ब्रत्रक के मामलों पर चचाट की थी।

27. असभकथन (A): आश्रम व्यवस्था उत्तर वैददक काल के दौरान शरू
ु हुई थी।

कारण (R): िोत्र की सींस्था प्रारीं सभक वैददक यि


ु के दौरान प्रकर् हुई थी।

उपरोतत के सन्दभट में तनम्नसलखखत में से कौन सा सही है

A. A और R दोनों सत्य हैं और R, A की सही व्याख्या करता है ।

B. A और R दोनों सत्य हैं लेककन R, A की सही व्याख्या नहीीं है ।

C. A सच है लेककन R िलत है ।

Target PCS Lucknow Page 17


General Studies Paper#1-Sectional Paper#1

D. A िलत है लेककन R सत्य है

उत्तर. C

स्पष्र्ीकरण: िोत्र की उत्पपत्त ऋग्वैददक काल में हुई थी. इसका शाक्ब्दक अथट िौशाला या अपनी िायों
को रखने का स्थान है । वैददक काल में िाय को सबसे महत्वपण
ू ट िन माना जाता था और एक-दस
ू रे
से घतनष्ठ रूप से जड
ु े लोि उन्हें एक ही स्थान पर रखते थे।

28. असभकथन (A): प्रारीं सभक वैददक काल के दौरान उनके अगिकाींश यद्
ु ि मवेसशयों के सलए लडे िए
थे।

कारण (R): शतपथ ब्राह्मण ग्रींथ जुताई की रस्मों के बारे में पवस्तार से बताता है ।

उपरोतत के सन्दभट में तनम्नसलखखत में से कौन सा सही है

A. A और R दोनों सत्य हैं और R, A की सही व्याख्या करता है ।

B. A और R दोनों सत्य हैं लेककन R, A की सही व्याख्या नहीीं है ।

C. A सच है लेककन R िलत है ।

D. A िलत है लेककन R सत्य है

उत्तर. B

स्पष्र्ीकरण: शतपथ ब्राह्मण शत


ु ल यजुवेद पर एक भाष्य है । यह भारतीय दशटन के पपता सींत
याज्ञवल्तय द्वारा सलखा िया है । ब्राह्मणों में सबसे पण
ू ,ट व्यवक्स्थत और महत्वपण
ू ट के रूप में वखणटत,
इसमें वैददक यज्ञ अनष्ु ठानों, प्रतीकवाद और पौराखणक कथाओीं की पवस्तत
ृ व्याख्या है । शतपथ
ब्राह्मण में जुताई से सींबगीं ित अनष्ु ठानों का उल्लेख है । भिवान बलराम को एक हल के साथ गचब्रत्रत
ककया िया है , जो खेती के महत्व का सझ
ु ाव दे ता है ।

29. तनम्नसलखखत कथनों को ध्यान से पढें :

1. ऋग्वैददक आयों ने अपने अगिकाींश यद्


ु ि िायों के सलए लडे।

2. ऋग्वैददक आयट मख्


ु यतः दे हाती लोि थे।

Target PCS Lucknow Page 18


General Studies Paper#1-Sectional Paper#1

3. ऋग्वैददक काल में भसू म सींपपत्त का मख्


ु य रूप थी।

उपरोतत में से कौन सा/से कथन सही है /हैं?

A. केवल 1

B. केवल 1 और 2

C. केवल 2 और 3

D. उपरोतत सभी

उत्तर. B

स्पष्र्ीकरण: वैददक काल या वैददक यि


ु , भारत के इततहास के अींततम काींस्य यि
ु और प्रारीं सभक लौह
यि
ु की अवगि है जब वेदों की रचना उत्तरी भारतीय उपमहाद्वीप में की िई थी, शहरी ससींिु घार्ी
सभ्यता के अींत और एक दस
ू रे शहरीकरण के बीच जो मध्य भारत-िींिा के मैदान में 600 ईसा पव
ू ट
में शरू
ु हुआ था। ऋग्वैददक समाज एक दे हाती समाज था और पशप
ु ालन उनका मख्
ु य व्यवसाय था।
तनिा मि
ु ा की इकाई थी, जो सोने की बनी होती थी। स्थानाींतररत कृपष का अभ्यास ककया जाता था
और जींिल के आवरण को जलाने के सलए आि का उपयोि ककया जाता था और कफर भसू म के साफ
पैच को बोया जाता था।

30. ऋग्वेद में वखणटत 'दस राजाओीं की लडाई' के बारे में तनम्नसलखखत कथनों को ध्यान से पढें :

1. यह यद्
ु ि एक ओर भरत और दस
ू री ओर दस सरदारों के बीच लडा िया था।

2. यह यद्
ु ि रावी नदी के समान परुषनी नदी के तर् पर लडा िया था।

3. इस यद्
ु ि ने राजा सद
ु ास को जीत ददलाई और भरत का वचटस्व स्थापपत ककया।

उपरोतत में से कौन सा/से कथन सही है /हैं?

A. केवल 1

B. केवल 1 और 2

C. केवल 2 और 3

D. उपरोतत सभी

Target PCS Lucknow Page 19


General Studies Paper#1-Sectional Paper#1

उत्तर. D

स्पष्र्ीकरण: दस राजाओीं की लडाई ब्रत्रत्सु पररवार के भरत राजा सद


ु ास और दस प्रससद्ि
जनजाततयों- परु
ु , यद,ु तव
ु स
ट ा, अन,ु िह्ु य,ु अलीना, पतत, भालान, सशव और पवशतनन के सींघ के बीच
थी। पव
ू ट पाींच को आयट जनजातत कहा जाता है जबकक बाद के 5 को िैर-आयट जनजातत कहा जाता
है । इस यद्
ु ि का वणटन ऋग्वेद के 7वें मींडल में चौपाई 18, 33 और 83 में समलता है । सद
ु ास दस
राजाओीं के परू े सींघ को हराने में सफल रहे और पवजयी हुए। सद
ु ास के सभी शत्रु पराक्जत हुए, हजारों
मारे िए।

जैन धर्व और बौद्ध धर्व

31. बौद्ि िमट में पततमोतखा (Patimokkha) का तया अथट है ?

A. राजा मेनेंडर के प्रचन

B. महायान बौद्ि िमट का वणटन

C. हीनयान बौद्ि िमट का वणटन

D. मठवासी जीवन के तनयम

उत्तर. D

स्पष्र्ीकरण: सभिुओीं और ननों (nuns) के मठवासी जीवन के सलए पततमोतखा सबसे महत्वपण
ू ट
सींदहता (code) है । यह उन कायों और भाषा को तनयींब्रत्रत करता है जो उनके मठवासी जीवन जीने के
सलए महत्वपण
ू ट हैं। यह उन राष्रीय कानन
ू ों की तरह काम करता है जो दे शों को तनयींब्रत्रत करते हैं।
यह अपरािों के साथ-साथ उनके दीं ड की स्पष्र् पररभाषा प्रदान करता है ।

32. बद्
ु ि ने तनम्नसलखखत में से ककस िणराज्य में महापररतनवाटण प्राप्त ककया?

A. सलच्छपव

B. मल्ल

C. पाला

Target PCS Lucknow Page 20


General Studies Paper#1-Sectional Paper#1

D. शातय

उत्तर. B

स्पष्र्ीकरण: मल्ल एक प्राचीन भारतीय िणराज्य था क्जसने प्राचीन भारत के सोलह महाजनपदों
(महान राज्यों) में से एक का िठन ककया था। मल्ल महावीर और िौतम बद्
ु ि की चुनी हुई मत्ृ यु
स्थान होने के सलए उल्लेखनीय है । कुशीनिर मल्ल िणराज्य की राजिानी थी। महात्मा बद्
ु ि को
कुशीनिर में महापररतनवाटण समला था।

33. बद्
ु ि चररत्र (Buddha Charita) के लेखक कौन थे?

A. नािाजन
ुट

B. अचवघोष

C. वससु मत्र

D. नािासेन

उत्तर. B

स्पष्र्ीकरण: बद्
ु िचररत्र अचवघोष द्वारा िौतम बद्
ु ि के जीवन पर सींस्कृत महाकाव्य शैली में एक
महाकाव्य कपवता है , क्जसकी रचना दस
ू री शताब्दी ईस्वी पव
ू ट में की िई थी। कपवता के 28 सिों में
से प्रथम 14 सींस्कृत में पवद्यमान हैं।

34. तनम्नसलखखत में से ककसके ससतकों पर बद्


ु ि का गचत्रण ककया िया है ?

A. कतनष्क

B. पवम कडकफसेस

C. बद्
ु ििप्ु त

D. नहपान

उत्तर. A

Target PCS Lucknow Page 21


General Studies Paper#1-Sectional Paper#1

स्पष्र्ीकरण: कतनष्क ने बद्


ु ि को सम्मातनत करने वाले ससतके जारी ककए थे । कुषाण सम्रार्
कतनष्क का ससतका, बद्
ु ि की तस्वीर के साथ और पीछे ग्रीक सलपप में उनका नाम "बोड्डो" सलखा है ,
लिभि 120 आम यि
ु (CE) में खनन ककया िया था।

35. कल्पसत्र
ू के लेखक कौन थे?

A. हे मचींि

B. वससु मत्र

C. स्थूलभि

D. भिबाहु

उत्तर. D

स्पष्र्ीकरण: कल्प सत्र


ू एक जैन ग्रींथ है क्जसमें जैन तीथंकरों की आत्मकथाएाँ हैं। भिबाहु को पाठ का
लेखक माना जाता है और पारीं पररक रूप से कहा जाता है कक इसकी रचना महावीर के तनवाटण
(Nirvana) के लिभि एक सौ पचास साल बाद हुई थी।

36. सच
ू ी I को सच
ू ी II से सम
ु ेसलत कीक्जए और नीचे ददए िए कूर् से सही उत्तर का चयन कीक्जए:

सच
ू ी I सच
ू ी II

(पात्र) (सींबगीं ित वस्तए


ु )ीं

a. बोगिसत्व मैत्रय
े 1. कत्योत्सिट आसन

b. िौतम बद्
ु ि 2. कुप्पी

c. जीना 3. िमटचि प्रवतटन मि


ु ा:

d. तीथंकर 4. िरुडध्वज मैतय


े :

कोड:

a b c d

Target PCS Lucknow Page 22


General Studies Paper#1-Sectional Paper#1

A. 4 3 2 1

B. 3 2 4 1

C. 2 3 4 1

D. 3 4 1 2

उत्तर. C

स्पष्र्ीकरण: कई बोगिसत्व हैं:- मैत्रय


े भपवष्य के बद्
ु ि हैं, क्जनका जन्म अिले यि
ु में ज्ञान की
सशिा दे ने के सलए होिा। बोगिसत्व मैत्रय
े की पहचान उनके बाएीं हाथ में रखे छोर्े कुप्पी से होती है ,
जो अमरता के तरल से भरा होता है । मैत्रय
े सबसे शरु
ु आती बोगिसत्व हैं क्जनके चारों ओर एक पींथ
पवकससत हुआ और तीसरी शताब्दी से शास्त्रों में इसका उल्लेख ककया िया है । उन्हें बौद्ि िमट के
सभी स्कूलों द्वारा स्वीकार ककया िया था और अभी भी एकमात्र बोगिसत्व है क्जसे आमतौर पर
थेरवाद परीं परा द्वारा सम्मातनत ककया जाता है । जैन िमट में , तीथंकर अररहीं त हैं जो केवलज्ञान (शद्
ु ि
अनींत ज्ञान) प्राप्त करने के बाद सच्चे िमट का प्रचार करते हैं। एक अररहीं त को जीना (पवजेता) भी
कहा जाता है , क्जसने िोि, मोह, अहीं कार और लोभ जैसे आींतररक शत्रओ
ु ीं पर पवजय प्राप्त की है ।
कायोत्सिट एक योि मि
ु ा है जो जैन ध्यान का एक महत्वपण
ू ट अींि है । इसका शाक्ब्दक अथट है "शरीर
को खाररज करना"। एक तीथंकर को या तो योि मि
ु ा में बैठे हुए या कायोत्सिट मि
ु ा में खडे होने का
प्रतततनगित्व ककया जाता है ।

37. असभकथन (A): मेनेंडर ने बौद्ि सशिक नािसेन के साथ क्जन सवालों और जवाबों का आदान-
प्रदान ककया, क्जन्हें नािाजन
ुट भी कहा जाता है ; समसलींदपन्हो पस्
ु तक का पवषय है ।

कारण (R): हे सलयोडोरस ने दस


ू री शताब्दी ईसा पव
ू ट के मध्य में मध्य प्रदे श में पवददसा (पवददसा क्जले
का मख्
ु यालय) के पास वासद
ु े व के सम्मान में एक स्तींभ की स्थापना की।

उपरोतत के सन्दभट में तनम्नसलखखत में से कौन सा सही है

A. A और R दोनों सत्य हैं और R, A की सही व्याख्या करता है ।

B. A और R दोनों सत्य हैं लेककन R, A की सही व्याख्या नहीीं है ।

C. ए सच है लेककन आर झूठा है ।

D. A िलत है लेककन R सत्य है

Target PCS Lucknow Page 23


General Studies Paper#1-Sectional Paper#1

उत्तर. B

स्पष्र्ीकरण: समसलींद पन्हा एक बौद्ि ग्रींथ है जो 100 ईसा पव


ू ट और 200 ईस्वी के बीच का है । यह
बौद्ि ऋपष नािसेन और बैक्तरया के इींडो-ग्रीक राजा मेनेंडर प्रथम के बीच एक सींवाद का एक ररकॉडट
है , क्जन्होंने दस
ू री शताब्दी ईसा पव
ू ट में शासन ककया था। हे सलयोडोरस स्तींभ एक पत्थर का स्तींभ है
क्जसे लिभि 113 ईसा पव
ू ट मध्य भारत में बेसनिर (पवददशा, मध्य प्रदे श के पास) में बनाया िया
था। स्तींभ को िरुड कहा जाता था क्जसे हे सलयोडोरस द्वारा बनाया िया था, और यह िरुड दे वता के
बारे में है । हे सलयोडोरस खद
ु को एक भािवत के रूप में वखणटत करता है , जो भिवान वासद
ु े व कृष्ण
का उपासक है और उसने इस भिवान के सम्मान में इस स्तींभ की स्थापना की थी। िरुड पिी को
पवष्णु का वाहन माना जाता है ।

38. असभकथन (A): अशोक ने मदहलाओीं सदहत पवसभन्न सामाक्जक समह


ू ों के बीच अपने िम्म के
प्रचार के सलए िम्म महा मत रास तनयत
ु त ककया था।

कारण (R): तीसरी बौद्ि सींिीतत (सींिीतत) अशोक के पास थी और समशनररयों को न केवल दक्षिण
भारत बक्ल्क श्रीलींका, बमाट और अन्य दे शों में भी भेजा िया था।

उपरोतत के सन्दभट में तनम्नसलखखत में से कौन सा सही है

A. A और R दोनों सत्य हैं और R, A की सही व्याख्या करता है ।

B. A और R दोनों सत्य हैं लेककन R, A की सही व्याख्या नहीीं है ।

C. A सच है लेककन R झठ
ू ा है ।

D. A िलत है लेककन R सत्य है

उत्तर. B

स्पष्र्ीकरण: एक महामात्रा (क्जसका अथट है ("उच्च पद का अगिकारी") भारतीय सम्रार् अशोक द्वारा
स्थापपत एक "नैततकता का अगिकारी" था (शासनकाल 269-233BCE)। िम्म महामत्ता अगिकाररयों
का एक िठन था जो िम्म का प्रचार सतु नक्चचत करने और जनता की राय के बारे में सम्रार् को
जािरूक रखने के दोहरे कायों की सेवा करता था। उन्होंने कैददयों के कल्याण के साथ-साथ पवसभन्न
सींप्रदायों के लोिों के साथ उगचत व्यवहार भी दे खा। तीसरी बौद्ि पररषद पार्सलपत्र
ु के अशोकराम में
लिभि 240 ईसा पव
ू ट में बल
ु ाई िई थी, क्जसे सम्रार् अशोक के सींरिण में माना जाता था। पररषद
की अध्यिता मोिलीपत्त
ु ततस्सा ने की थी।

Target PCS Lucknow Page 24


General Studies Paper#1-Sectional Paper#1

39. तनम्नसलखखत कथनों पर पवचार करें :

1. जैन स्थूलभि के नेतत्ृ व में दक्षिण भारत िए।

2. िौतम बद्
ु ि ने सारनाथ में अपना पहला उपदे श ददया था।

3. चींििप्ु त जैन िमट को स्वीकार करने के सलए भिबाहु से प्रभापवत हुए थे।

उपरोतत में से कौन सा/से कथन सही है /हैं?

A. केवल 1

B. केवल 1 और 2

C. केवल 2 और 3

D. उपरोतत सभी

उत्तर. C

स्पष्र्ीकरण: तीसरी या चौथी शताब्दी ईसा पव


ू ट में मौयट साम्राज्य में 12 साल के अकाल के दौरान
चवेताींबर जैन शाखा के सींस्थापक स्थल
ू भि थे। वह भिबाहु और सींभत
ु पवजय के सशष्य थे। उनके पपता
सकर्ला थे जो चींििप्ु त मौयट के आने से पहले नींद राज्य में मींत्री थे। जैन िमट के ददिींबर सींप्रदाय के
अनस
ु ार, आचायट भिबाहु जैन िमट में अींततम श्रुत केवसलन थे, लेककन चवेताींबर का मानना है कक
अींततम श्रुत केवसलन आचायट स्थूलभि थे, लेककन भिबाहु ने इसे बताने से मना ककया था। वे
अपवभाक्जत जैन सींघ के अींततम आचायट थे।

40. तनम्नसलखखत कथनों को ध्यानपव


ू क
ट पदढए और नीचे ददए िए कूर् का प्रयोि कर सही उत्तर
चतु नए:

1. पाचवटनाथ तेईसवें तीथंकर थे और वे बनारस के थे।

2. विटमान महावीर का जन्म वैशाली के पास एक िााँव में हुआ था।

3. महावीर की माता सलच्छपव प्रमख


ु चेतक की पत्र
ु ी थी।

4. ब्रह्मचयट ससद्िाींत महावीर द्वारा जोडा िया था।

Target PCS Lucknow Page 25


General Studies Paper#1-Sectional Paper#1

उपरोतत में से कौन सा/से कथन सही है /हैं?

A. केवल 1 और 2

B. केवल 1, 2 और 3

C. केवल 2, 3 और 4

D. उपरोतत सभी

उत्तर. D

स्पष्र्ीकरण: महावीर, क्जन्हें विटमान भी कहा जाता है , जैन िमट के 24वें तीथंकर हैं। वे 23वें तीथंकर
पाचवटनाथ के आध्याक्त्मक उत्तरागिकारी थे। महावीर का जन्म छठी शताब्दी ईसा पव
ू ट के शरु
ु आती
भाि में ब्रबहार, भारत में एक शाही पररवार में हुआ था। उनकी माता का नाम ब्रत्रशला और उनके
पपता का नाम ससद्िाथट था।

र्हाजनपि और र्गध का उिय

41. तनम्नसलखखत में से कौन सा िेत्र अींि महाजनपदों से जुडा था?

A. ब्रबहार के मि
ींु ेर और भािलपरु के आितु नक क्जले

B. पर्ना, िया और शाहाबाद के कुछ दहस्सों के आितु नक क्जले

C. पव
ू ी यप
ू ी के दे वररया, बस्ती, िोरखपरु और ससद्िाथटनिर के आितु नक क्जले

D. उपरोतत सभी

उत्तर. A

स्पष्र्ीकरण: अींि एक प्राचीन भारतीय साम्राज्य था जो पव


ू ी भारतीय उपमहाद्वीप और सोलह
महाजनपदों ("बडे राज्य") में से एक पर पवकससत हुआ था। यह अपने पडोसी और प्रततद्वींद्वी मिि
के पव
ू ट में क्स्थत था, और यह चींपा नदी द्वारा मिि से अलि हो िया था, जो आितु नक समय में
ब्रबहार राज्य के भािलपरु और मि
ींु ेर में है ।

Target PCS Lucknow Page 26


General Studies Paper#1-Sectional Paper#1

42. जब ससकींदर ने भारत पर आिमण ककया, तो मिि के शासक कौन थे?

A. हयंकासी

B. शीसशनािस

C. नींद

D. मौयट

उत्तर. C

स्पष्र्ीकरण: नींद वींश ने भारतीय उपमहाद्वीप के उत्तरी भाि में चौथी शताब्दी ईसा पव
ू ट के दौरान
और सींभवतः 5 वीीं शताब्दी ईसा पव
ू ट के दौरान शासन ककया था। नींदों ने पव
ू ी भारत के मिि िेत्र में
शशन
ु ाि वींश को उखाड फेंका, और उत्तरी भारत के एक बडे दहस्से को शासमल करने के सलए अपने
साम्राज्य का पवस्तार ककया। एक महत्वाकाींिी यव
ु क चींििप्ु त मौयट ने मिि पर हमला ककया और
उसे जीत सलया। वह नींद शासन का अींत था।

43. तनम्नसलखखत में से कौन मिि का सींस्थापक था?

A. हयंक और बह
ृ िथ

B. अजातशत्रु और जरासींि

C. उदतयन नद अजशत्रु

D. जरासींि और बह
ृ िथ

उत्तर. D

स्पष्र्ीकरण: जरासींि और बह
ृ िथ मिि के सींस्थापक थे लेककन पवकास हयंक के तहत शरू
ु हुआ।

44. तनम्नसलखखत में से कौन सा महाजनपद राजतींत्रीय राज्य था?

A. मिि

B. कोसल

Target PCS Lucknow Page 27


General Studies Paper#1-Sectional Paper#1

C. वत्स

D. उपरोतत सभी

उत्तर. D

स्पष्र्ीकरण: िणताींब्रत्रक (republican) महाजनपदों पर ककसी राजा का शासन नहीीं था जबकक


राजशाही महाजनपदों पर एक राजा का शासन था। िणताींब्रत्रक महाजनपदों पर आम लोिों द्वारा चुने
िए लोिों के एक समह
ू का शासन था जबकक राजशाही महाजनपदों पर एक वींशानि
ु त राजा का
शासन था। मल्ल, वज्जी, कम्बोज और कुरु िणताींब्रत्रक राज्य थे जबकक मिि, कोसल, वत्स, आींर्ी,
अींि, काशी, िाींिार, शरू सेना, छे दी और मत्स्य प्रकृतत में राजशाही थे।

45. तनम्नसलखखत में से कौन सा बौद्ि ग्रींथ 16 महाजनपदों के नाम प्रदान करता है ?

A. जातक

B. समसम
ु गिरी के भाग्िा

C. अींित्त
ु रा तनकाय

D. मालपवकाक्ग्नसमत्रन

उत्तर. C

स्पष्र्ीकरण: अींित्त
ु र तनकाय जैसे प्राचीन बौद्ि ग्रींथों में सोलह महान राज्यों और िणराज्यों का बार-
बार उल्लेख समलता है जो भारतीय उपमहाद्वीप के पव
ू ी भाि में उत्तर-पक्चचम में िाींिार से लेकर
अींिा तक फैले एक िेत्र (belt) में पवकससत और फले-फूले थे।

46. सच
ू ी I को सच
ू ी II से सम
ु ेसलत कीक्जए और नीचे ददए िए कूर् से सही उत्तर का चयन कीक्जए:

सच
ू ी I सच
ू ी II

(शासक) (महाजनपद)

a. प्रसेनक्जता 1. वत्स

b. उदयन 2. अवींती

Target PCS Lucknow Page 28


General Studies Paper#1-Sectional Paper#1

c. प्रदयोता 3. मिि

d. अजातशत्रु 4. कोसल

कोड:

a b c d

A. 4 3 2 1

B. 4 1 2 3

C. 2 3 4 1

D. 3 4 1 2

उत्तर. B

स्पष्र्ीकरण: महाजनपद प्राचीन भारत में मौजद


ू सोलह राज्यों का एक समह
ू था। यह सब तब शरू

हुआ जब उत्तर वैददक काल की जनजाततयों (जनों) ने अपने स्वयीं के िेत्रीय समद
ु ायों को बनाने का
फैसला ककया, क्जसने अींततः 'राज्य' या 'जनपद' नामक बक्स्तयों के नए और स्थायी िेत्रों को जन्म
ददया।

47. असभकथन (A): सलच्छपवयों की स्वतींत्रता को नष्र् करने और यद्


ु ि में उन्हें हराने के सलए
अजातशत्रु को 16 साल लि िए।

कारण (R): मिि के मींत्री वरसाकार का उपयोि वैशाली के सलच्छपवयों में मतभेद के बीज बोने के
सलए ककया जाता था।

उपरोतत के सन्दभट में तनम्नसलखखत में से कौन सा सही है

A. A और R दोनों सत्य हैं और R, A की सही व्याख्या करता है ।

B. A और R दोनों सत्य हैं लेककन R, A की सही व्याख्या नहीीं है ।

C. A सच है लेककन R िलत है ।

D. A िलत है लेककन R सत्य है

उत्तर. A

Target PCS Lucknow Page 29


General Studies Paper#1-Sectional Paper#1

स्पष्र्ीकरण: ब्रबींब्रबसार ने लिभि 544 ई.प.ू से से 492 ई.प.ू लिभि 52 वषों तक शासन ककया।
उसका उत्तरागिकारी उसका पत्र
ु अजातशत्रु (492-460 ई.प.ू ) हुआ। अजातशत्रु ने अपने पपता को मार
डाला और ससींहासन पर कब्जा कर सलया। अपने परू े शासनकाल में उन्होंने पवस्तार की आिामक
नीतत अपनाई। मिि और कोशल के बीच एक लींबे समय तक सींघषट शरू
ु हुआ। अींततः अजातशत्रु को
यद्
ु ि में सवटश्रेष्ठ समला, और कोशलन राजा को अपनी बेर्ी को अजातशत्रु से पववाह करके और उसे
काशी के एकमात्र अगिकार में छोडकर शाींतत के सलए मजबरू होना पडा। हालााँकक उनकी मााँ एक
सलच्छवी राजकुमारी थीीं, लेककन इसने उन्हें वैशाली के खखलाफ यद्
ु ि करने से नहीीं रोका। उन्होंने
सलच्छपवयों में मतभेद पैदा ककए और अींत में उनके िेत्र पर आिमण करके और उन्हें यद्
ु ि में
हराकर उनकी स्वतींत्रता को नष्र् कर ददया। वैशाली को नष्र् करने में उसे परू े 16 साल लिे। मींत्री
वरसाकार वैशाली के सलच्छपवयों में मतभेद के बीज बोने में सफल रहे , क्जससे अजातशत्रु िणतींत्र को
जीत सके।

48. असभकथन (A): मौयट काल के दौरान, पपींडकारा िाींवों के समह


ू से सींयत
ु त रूप से एकत्र ककया
िया कर (tax) था।

कारण (R): दहरण्य एक ऐसा कर था क्जसका भि


ु तान वस्तु के रूप में नहीीं बक्ल्क नकद में ककया
जाता था।

उपरोतत के सन्दभट में तनम्नसलखखत में से कौन सा सही है

A. A और R दोनों सत्य हैं और R, A की सही व्याख्या करता है ।

B. A और R दोनों सत्य हैं लेककन R, A की सही व्याख्या नहीीं है ।

C. A सच है लेककन R िलत है ।

D. A िलत है लेककन R सत्य है

उत्तर. B

स्पष्र्ीकरण: पपींडकारा करों का ढे र है । इसने एक िााँव या िााँवों के समह


ू से सींयत
ु त रूप से एकब्रत्रत
कर का सझ
ु ाव ददया। कोषािार चरवाहों और पशप
ु ालकों पर पशओ
ु ीं की सींख्या और उपज पर कर
लिाने का हकदार था। दहरण्य एक ऐसा कर था क्जसका भि
ु तान वस्तु के रूप में नहीीं बक्ल्क नकद
में ककया जाता था।

Target PCS Lucknow Page 30


General Studies Paper#1-Sectional Paper#1

49. मिि के तनम्नसलखखत राजवींशों को सही कालानि


ु समक िम में व्यवक्स्थत करें :

1. सशशन
ु ाि

2. हयंका (Haryanka)

3. नींद

4. मौयट

नीचे ददए िए कूर् का प्रयोि कर सही उत्तर चुतनए:

A. 1–3–4–2

B. 2-3–1–4

C. 1-2–3–4

D. 2–1–3–4

उत्तर. D

स्पष्र्ीकरण: मिि साम्राज्य ने भारत में 684 ईसा पव


ू ट - 320 ईसा पव
ू ट तक शासन ककया। ब्रबींब्रबसार,
(जन्म 543 - मत्ृ यु 491 ईसा पव
ू )ट , मिि के भारतीय साम्राज्य के शरु
ु आती राजाओीं में से एक था।
माना जाता है कक राज्य के उनके पवस्तार, पवशेष रूप से पव
ू ट में अींि के राज्य के उनके पवलय ने
मौयट साम्राज्य के बाद के पवस्तार की नीींव रखी। मिि साम्राज्य का उल्लेख दो महान महाकाव्यों
रामायण और महाभारत में समलता है । मिि साम्राज्य पर 544 ईसा पव
ू ट से 322 ईसा पव
ू ट तक
शासन करने वाले तीन राजवींश थे। पहला हयंका राजवींश (544 ईसा पव
ू ट से 412 ईसा पव
ू )ट , दस
ू रा
सशशन
ु ाि राजवींश (412 ईसा पव
ू ट से 344 ईसा पव
ू )ट था और तीसरा नींद वींश (344 ईसा पव
ू -ट 322 ईसा
पव
ू )ट था। िनानींद नींद वींश के अींततम राजा थे। ग्रीक ग्रींथों में उन्हें एग्राम्स या जाींड्रामेस के रूप में
जाना जाता है । उसके शासन काल में ही ससकींदर ने भारत पर आिमण ककया था। 322 ईसा पव
ू ट में
चींििप्ु त मौयट ने िनानींद को उखाड फेंका, क्जन्होंने मौयट वींश नामक मिि के एक नए शासक वींश
की स्थापना की।

50. तनम्नसलखखत कथनों को ध्यानपव


ू क
ट पदढए और नीचे ददए िए कूर् का प्रयोि कर सही उत्तर
चुतनए:

1. पतत की मत्ृ यु के बाद पविवा के सती का पहला उदाहरण मौयट काल का है ।

Target PCS Lucknow Page 31


General Studies Paper#1-Sectional Paper#1

2. िप्ु ताओीं द्वारा जारी ककए िए सोने के ससतकों का नाम दीनार था।

3. कुमारमात्य िप्ु त काल में क्जले के मखु खया थे।

ऊपर ददए िए कथनों में से कौन-सा/से िलत है /हैं?

A. केवल 1

B. केवल 1 और 3

C. केवल 2 और 3

D. उपरोतत सभी

उत्तर. A

स्पष्र्ीकरण: सती प्रथा सददयों से भारतीय समाज का दहस्सा रही है और इसकी शरु
ु आत िप्ु त काल
के समय से हुई है । पतत की मत्ृ यु के बाद पविवा के बसलदान का पहला उदाहरण िप्ु त काल का है ।
शाही िप्ु ताओीं ने अच्छी तरह से ढाले और अच्छी तरह से तनष्पाददत सोने के ससतके जारी ककए। ये
ससतके, क्जन्हें दीनार भी कहा जाता है , ज्यादातर उत्तर भारत में पाए जाते थे।

र्ौयव िाम्राज्य

51. चींििप्ु त मौयट के दरबार में यन


ू ानी राजदत
ू कौन था?

A. मेिास्थनीज

B. सेल्यक
ू ोस

C. मेनेंडर

D. डेमेदरयस

उत्तर. A

स्पष्र्ीकरण: मेिस्थनीज का जन्म एसशया माइनर में हुआ था और वह भारत के पार्सलपत्र


ु में
चींििप्ु त मौयट के सेल्यसू सड वींश के सेल्यक
ू स I तनकेर्र के राजदत
ू बने।

Target PCS Lucknow Page 32


General Studies Paper#1-Sectional Paper#1

52. बौद्ि िमट अपनाने के सलए अशोक को प्रभापवत करने वाला सभिु था।

A. पवष्णु िप्ु त

B. उपिप्ु त

C. ब्रह्मा िप्ु त

D. बह
ृ िथ

उत्तर. B

स्पष्र्ीकरण: उपिप्ु त (तीसरी शताब्दी ईसा पव


ू )ट एक बौद्ि सभिु थे। सींस्कृत ग्रींथ अशोकवदन की
कुछ कहातनयों के अनस
ु ार वे मौयट सम्रार् अशोक के आध्याक्त्मक िरु
ु थे। उपिप्ु त के सशिक सनवासी
थे जो बद्
ु ि के पररचारक आनींद के सशष्य थे।

53. मौयट काल में स्थातनक (स्थातनक) कौन था?

A. क्जला प्रशासक

B. प्राींतीय प्रशासक

C. ग्राम प्रशासक

D. निर प्रशासक

उत्तर. A

स्पष्र्ीकरण: स्थातनक मौयट साम्राज्य में क्जला प्रशासन थे। प्रशासन के उद्दे चयों के सलए प्राींतों को
क्जलों में पवभाक्जत ककया िया था और अगिकाररयों का एक समह
ू प्रत्येक क्जले में काम करता था।

54. तनम्नसलखखत में से ककस एक ने मौयट समाज के सात विों में पवभाजन का उल्लेख ककया?

A. अशोक के सशलालेख

B. इींडडका

C. कौदर्ल्य का अथटशास्त्र

Target PCS Lucknow Page 33


General Studies Paper#1-Sectional Paper#1

D. पवष्णु परु ाण

उत्तर. B

स्पष्र्ीकरण: इींडडका भारत में यन


ू ानी राजदत
ू मेिस्थनीज द्वारा लिभि 300 ईसा पव
ू ट में सलखी िई
पस्
ु तक है । मेिस्थनीज के 'इींडडका' में मौयट समाज के सात विों में पवभाजन का पवशेष रूप से
उल्लेख ककया िया था। भारतीयों को आम तौर पर सात श्रेखणयों में पवभाक्जत ककया िया था-
बद्
ु गिमान परु
ु ष (ब्राह्मण), ककसान, चरवाहे, कारीिर, सैतनक, दे खने वाले और सेना और नौसेना के
अगिकाररयों सदहत सरकारी अगिकारी।

55. ककस घर्ना ने अशोक की प्रशासतनक नीतत में िहरा पररवतटन लाया?

A. तीसरी बौद्ि पररषद

B. कसलींि यद्
ु ि

C. उनका बौद्ि िमट ग्रहण करना

D. उनका समशनरी को सीलोन भेजना

उत्तर. B

स्पष्र्ीकरण: कसलींि यद्


ु ि (262 ईसा पव
ू ट समाप्त हुआ) अशोक के अिीन मौयट साम्राज्य और पव
ू ी
तर् पर क्स्थत एक स्वतींत्र सामींती साम्राज्य कसलींि राज्य के बीच लडा िया था जो वतटमान में
ओडडशा राज्य और आींध्र प्रदे श के उत्तर में है । कसलींि यद्
ु ि में भारतीय इततहास की सबसे बडी और
सबसे खन
ू ी लडाइयों में से एक शासमल थी। कसलींिों के अप्रत्यासशत रूप से भयींकर प्रततरोि के
बावजूद, मौयट साम्राज्य ने जीत का दावा ककया और कसलींि राज्य पर कब्जा कर सलया। कसलींि का
कोई राजा नहीीं था तयोंकक यह साींस्कृततक रूप से ब्रबना ककसी राजा के चलाया जाता था। यह एकमात्र
प्रमख
ु यद्
ु ि है जो अशोक ने ससींहासन पर बैठने के बाद लडा था। कहा जाता है कक इस यद्
ु ि के
रततपात ने अशोक को बौद्ि िमट अपनाने के सलए प्रेररत ककया था।

56. सच
ू ी I को सच
ू ी II से सम
ु ेसलत कीक्जए और नीचे ददए िए कूर् से सही उत्तर का चयन कीक्जए:

सच
ू ी I सच
ू ी II

(सशलालेख) (पवषय)

Target PCS Lucknow Page 34


General Studies Paper#1-Sectional Paper#1

a. सशलालेख 1 1. पशु बसल को प्रततबींगित करता है

b. सशलालेख 2 2. सामाक्जक कल्याण के उपायों को दशाटता है

c. सशलालेख 3 3. ब्राह्मणों का सम्मान

d. सशलालेख 4 4. बडों का सम्मान

कोड:

a b c d

A. 4 3 2 1

B. 3 2 4 1

C. 1 2 3 4

D. 3 4 1 2

उत्तर. C

स्पष्र्ीकरण: जेम्स पप्रींसेप, एक ब्रब्रदर्श परु ातन और औपतनवेसशक प्रशासक, अशोक के सशलालेखों को
समझने वाले पहले व्यक्तत थे। अशोक के ये सशलालेख बौद्ि िमट के प्रथम मत
ू ट प्रमाण हैं। भारतीय
सम्रार् अशोक के प्रमख
ु सशलालेख अशोक के 14 अलि-अलि प्रमख
ु सशलालेखों का उल्लेख करते हैं
जो काफी पवस्तत
ृ हैं और ककसी भी भारतीय सम्रार् के सबसे परु ाने सशलालेखों में से कुछ का
प्रतततनगित्व करते हैं। ये सशलालेख कालानि
ु समक रूप से माइनर रॉक एडडतट्स (minor rock
edicts) से पहले के हैं।

57. असभकथन (A): “दे वी चींि िप्ु त” चींििप्ु त मौयट द्वारा ससींहासन के पररग्रहण की कहानी से
सींबगीं ित है ।

कारण (R): मि
ु ारािस इस बारे में है कक कैसे मौयट ने नींद राजा को उखाड फेंका।

उपरोतत के सन्दभट में तनम्नसलखखत में से कौन सा सही है

A. A और R दोनों सत्य हैं और R, A की सही व्याख्या करता है ।

B. A और R दोनों सत्य हैं लेककन R, A की सही व्याख्या नहीीं है ।

Target PCS Lucknow Page 35


General Studies Paper#1-Sectional Paper#1

C. A सच है लेककन R िलत है ।

D. A िलत है लेककन R सत्य है

उत्तर. D

स्पष्र्ीकरण: पवशाखदत्त ने अपने मि


ु ारािस में अतीत की राजनीततक घर्नाओीं का नार्क करने का
पवकल्प चुना, जो इस बात पर आिाररत है कक कैसे मौयट ने नींद राजा को उखाड फेंका। दे वी चींि
िप्ु ता चींि िप्ु त द्पवतीय (िप्ु त राजा) द्वारा सत्ता के सलए सींघषट पर आिाररत है ।

58. असभकथन (A): अशोक अपने सशलालेखों के माध्यम से लोिों से सीिे बात करने वाले पहले
भारतीय राजा थे, जो शाही आदे श दे ते थे।

कारण (R): अशोक के असभलेख सामान्यतः प्राकृत और ब्राह्मी सलपप में सलखे िए थे।

उपरोतत के सन्दभट में तनम्नसलखखत में से कौन सा सही है

A. A और R दोनों सत्य हैं और R, A की सही व्याख्या करता है ।

B. A और R दोनों सत्य हैं लेककन R, A की सही व्याख्या नहीीं है ।

C. A सच है लेककन R िलत है ।

D. A िलत है लेककन R सत्य है

उत्तर. A

स्पष्र्ीकरण: अशोक के सशलालेख भारत के मौयट साम्राज्य (322-185 ईसा पव


ू )ट के तीसरे राजा
अशोक (268-232 ईसा पव
ू )ट द्वारा स्तींभों, बडे पत्थरों और िफ
ु ा की दीवारों पर उत्कीणट 33 सशलालेख
हैं। अशोक के अगिकाींश सशलालेख प्राकृत की बोसलयों में हैं (उत्तर भारत में बोली जाने वाली पवसभन्न
स्थानीय भाषाएीं, सादहक्त्यक सींस्कृत से अलि, लेककन सींबगीं ित हैं), जो कई शताक्ब्दयों तक
उपमहाद्वीप में राजनीततक शक्तत की भाषा थी, और प्रारीं सभक रूप में ब्राह्मी सलपप में सलखे िए हैं।

59. तनम्नसलखखत कथनों पर पवचार करें :

1. मेिस्थनीज चींििप्ु त मौयट के दरबार में आया था।

Target PCS Lucknow Page 36


General Studies Paper#1-Sectional Paper#1

2. मेिस्थनीज की इींडडका मौयट प्रशासन के बारे में जानकारी प्रदान करती है ।

3. 'एररगियन सािर का पेररप्लस' दस


ू री शताब्दी ईस्वी में प्राचीन भारतीय भि
ू ोल और वाखणज्य के
अध्ययन के सलए मल्
ू यवान तथ्य (data) प्रदान करता है ।

4. फाह्यान ने िप्ु त काल में भारत की सामाक्जक, िासमटक और आगथटक क्स्थततयों का वणटन ककया।
उपरोतत में से कौन सा/से कथन सही है /हैं?

A. केवल 1 और 2

B. केवल 1 और 4

C. केवल 2, 3 और 4

D. उपरोतत सभी

उत्तर. D

स्पष्र्ीकरण: मेिस्थनीज एक प्राचीन यन


ू ानी इततहासकार, राजनतयक थे। वह पार्सलपत्र
ु में चींििप्ु त
मौयट के सेल्यक
ू स I तनकेर्र (Seleucus I Nicator) के राजदत
ू बने। उन्होंने इींडडका नामक पस्
ु तक
सलखी, जो मौयटकालीन भारत का लेखा-जोखा है । एररगियन सािर का पेररप्लस दहींद महासािर में
व्यापार और पथ प्रदशटन के सलए एक रोमन काल की मािटदसशटका है ।

60. तनम्नसलखखत कथनों पर पवचार कीक्जएः

1. मिि की सबसे परु ानी राजिानी राजिीर में थी क्जसे उस समय गिररव्रज कहा जाता था।

2. राजा उदतयन ने पार्सलपत्र


ु में एक नई राजिानी का तनमाटण ककया।

3. ससकींदर ने भारत पर अपने आिमण के दौरान नींदों को हराया था।

उपरोतत में से कौन सा/से कथन सही है /हैं?

A. केवल 1

B. केवल 1 और 2

C. केवल 2 और 3

D. उपरोतत सभी

Target PCS Lucknow Page 37


General Studies Paper#1-Sectional Paper#1

उत्तर. B

स्पष्र्ीकरण: 326 ईसा पव


ू ट में ससकींदर ने भारत पर आिमण ककया, ससींिु नदी को पार करने के बाद
वह तिसशला की ओर बढा. कफर उसने झेलम और गचनाब नददयों के बीच के राज्य के शासक राजा
पोरस को चन
ु ौती दी। हाइडस्पेश की लडाई 326 ईसा पव
ू ट में ससकींदर महान द्वारा राजा पोरस के
खखलाफ, हाइडस्पेश नदी के तर् पर लडी िई थी। लडाई के पररणामस्वरूप ग्रीक की जीत हुई। पौरव
उत्तर पक्चचमी भारतीय उपमहाद्वीप (वतटमान भारत और पाककस्तान) में एक प्राचीन भारतीय राजवींश
थे, क्जसके राजा पोरस थे।

ििंगर् युग

61. 'पददत्रप्ु पट्


ु र्ु' तनम्नसलखखत में से ककस राजा की स्ततु त में कपवताओीं का सींग्रह है ?

A. पाींड्या

B. चेर

C. चोल

D. इनमें से कोई नहीीं

उत्तर. B

स्पष्र्ीकरण: जबकक पट्र्ुपट्र्ू 108 छीं दों में चेर राजाओीं की मदहमा तक सीसमत है , परु ाणनरु
ु में तीन
सौ तनन्यानवे कपवताओीं में पवषयों का विीकरण है ।

62. सींिम काल के प्रमख


ु दे वता कौन थे?

A. ससयोन

B. पशप
ु तत

C. सशव

D. राम

Target PCS Lucknow Page 38


General Studies Paper#1-Sectional Paper#1

उत्तर. A

स्पष्र्ीकरण: सींिम काल के प्राथसमक दे वता ससयोन या मरु


ु िन थे, जो आज भी तसमलों के प्राथसमक
दे वता हैं। उन्हें अरुपदाई वीद ु के नाम से जाने वाले छह तनवासों से सम्मातनत ककया िया था।
इससलए, सींिम यि
ु से लेकर आज तक मरु
ु िा पज
ू ा तसमल समाज और सींस्कृतत का आिार है ।

63. तनम्नसलखखत में से कौन तसमल कपवयों की एक सभा सींिम के सींरिक थे?

A. चोल

B. कदीं बी

C. पींड्या

D. चेर

उत्तर. C

स्पष्र्ीकरण: सींिम यि
ु दक्षिण भारत में पहली शताब्दी ईसा पव
ू ट से दस
ू री शताब्दी आम यि
ु के अींत
तक की अवगि है । तसमल ककीं वदीं ततयों का कहना है कक प्राचीन दक्षिण भारत में 3 सींिम (तसमल
कपवयों की अकादमी) आयोक्जत ककए िए थे, क्जन्हें मच
ु चींिम कहा जाता है । ये सींिम मदरु ै के पाींड्य
राजाओीं के शाही सींरिण में फले-फूले। सींिम यि
ु के दौरान शाससत तीन राजवींश चेर, चोल और
पाींड्य थे।

64. 'तोलकाक्प्पयम' ककससे सींबगीं ित है ?

A. प्रशासन

B. कानन

C. व्याकरण

D. इनमें से कोई नहीीं

उत्तर. C

Target PCS Lucknow Page 39


General Studies Paper#1-Sectional Paper#1

स्पष्र्ीकरण: तोलकाक्प्पयम सबसे प्राचीन तसमल व्याकरण पाठ है और तसमल सादहत्य का सबसे
परु ाना मौजद
ू ा लींबा काम है । तोलकाक्प्पयम की जीपवत पाींडुसलपपयों में तीन पस्
ु तकें हैं, क्जनमें से
प्रत्येक में नौ अध्याय हैं, क्जसमें नरू पा मीर्र में कुल 1,612 सत्र
ू हैं।

65. कदीं ब राजाओीं की राजिानी थी

A. काींची

B. तींजौर

C. बादामी

D. बनवासी

उत्तर. D

स्पष्र्ीकरण: कदीं ब साम्राज्य सातवाहनों के बाद कनाटर्क के उत्तरी भाि पर शासन करने वाला पहला
कन्नड राजवींश था। मयरू वमाट उस साम्राज्य का प्रथम शासक था। बनवासी कन्नड साम्राज्य कदीं ब की
प्राचीन राजिानी थी।

66. सच
ू ी I को सच
ू ी II से सम
ु ेसलत कीक्जए और नीचे ददए िए कूर् से सही उत्तर का चयन कीक्जए:

सच
ू ी I सच
ू ी II

(सींवत) (अवगि)

a. शक सींवत 1. 606 ई

b. पविम सींवत 2. 319 ई

c. िप्ु त सींवत 3. 57 ई.पू

d. हषट सींवत 4. 78 ई

कोड:

a b c d

Target PCS Lucknow Page 40


General Studies Paper#1-Sectional Paper#1

A. 4 3 2 1

B. 3 1 4 2

C. 2 3 4 1

D. 3 4 1 2

उत्तर. A

स्पष्र्ीकरण: माना जाता है कक शक यि


ु की स्थापना शतवाहन वींश के राजा शासलवन्नान ने की थी।
शक कैलेंडर में 365 ददन और 12 महीने होते हैं जो ग्रेिोररयन कैलेंडर की सींरचना के समान है ।
पविम सींवत, क्जसे ब्रबिम सींवत या पविमी कैलेंडर के रूप में भी जाना जाता है , भारतीय
उपमहाद्वीप और नेपाल में उपयोि ककया जाने वाला ऐततहाससक दहींद ू कैलेंडर है । पारीं पररक पविम
सींवत कैलेंडर चींि मास और सौर नित्र वषट का उपयोि करता है । िप्ु त यि
ु एक ऐततहाससक कैलेंडर
यि
ु है जो 318 - 319 ई. से शरू
ु होता है । इसका उपयोि िप्ु त सम्रार्ों, साथ ही साथ उनके
जािीरदारों और वतटमान उत्तर भारत और नेपाल में उनके उत्तरागिकाररयों द्वारा ककया जाता था।
हषटविटन ने हषट सींवत को 606 ईस्वी में ससींहासन पर बैठते समय पेश ककया। 606 ईस्वी वह
महाकाव्य वषट था जब हषटविटन अपने बडे भाई की मत्ृ यु के बाद ससींहासन पर बैठा था, क्जसे सींसाका
द्वारा मार ददया िया था।

67. असभकथन (A): सींिम यि


ु के दौरान मव
ु ेन्दर तीन प्रमख
ु थे।

कारण (R): ततनाई का तात्पयट तसमल िेत्र के पाररक्स्थततक विीकरण से है ।

उपरोतत के सन्दभट में तनम्नसलखखत में से कौन सा सही है

A. A और R दोनों सत्य हैं और R, A की सही व्याख्या करता है ।

B. A और R दोनों सत्य हैं लेककन R, A की सही व्याख्या नहीीं है ।

C. A सच है लेककन R िलत है ।

D. A िलत है लेककन R सत्य है

उत्तर. B

Target PCS Lucknow Page 41


General Studies Paper#1-Sectional Paper#1

स्पष्र्ीकरण: सींिम कपवताओीं में मव


ु ें ि का उल्लेख है । यह एक तसमल शब्द है , क्जसका अथट है तीन
प्रमख
ु , तीन शासक पररवारों के मखु खया, चोल, चेर और पाींड्य के सलए उपयोि ककया जाता है ।
मखु खया तनयसमत कर (tax) नहीीं वसल
ू ते थे। इसके बजाय, उन्होंने लोिों से उपहार माींिे और प्राप्त
ककए। ततनाई तसमल िेत्र के पाररक्स्थततक विीकरण को सींदसभटत करता है ।

68. असभकथन (A): दस


ू री शताब्दी ईसा पव
ू ट के मध्य में , एलालन नामक एक चोल राजा ने श्रीलींका
पर पवजय प्राप्त की और लिभि 50 वषों तक इस पर शासन ककया।

कारण (R): दस
ू री शताब्दी ईस्वी में चोल राजा कररकाल ने पह
ु ार की स्थापना की और श्रीलींका से
लाए िए यद्
ु ि के 12,000 कैददयों द्वारा प्रदान ककए िए श्रम का उपयोि करके कावेरी नदी के
ककनारे एक तर्बींि का तनमाटण ककया।

उपरोतत के सन्दभट में तनम्नसलखखत में से कौन सा सही है

A. A और R दोनों सत्य हैं और R, A की सही व्याख्या करता है ।

B. A और R दोनों सत्य हैं लेककन R, A की सही व्याख्या नहीीं है ।

C. A सच है लेककन R िलत है ।

D. A िलत है लेककन R सत्य है

उत्तर. B

स्पष्र्ीकरण: एल्लालन तसमल चोल वींश का सदस्य था, क्जसे "मनु तनगि चोलन" के नाम से भी
जाना जाता है , जो ससींहासन पर कब्जा करने के बाद 205 ईसा पव
ू ट से 161 ईसा पव
ू ट तक वतटमान
श्रीलींका में अनरु ािापरु साम्राज्य का राजा बन िया। चोल वींश दक्षिण भारत का एक तसमल
थैलासोिेदर्क (thalassocratic) साम्राज्य था, जो दतु नया के इततहास में सबसे लींबे समय तक शासन
करने वाले राजवींशों में से एक था। चोल का सबसे परु ाना सींदभट मौयट साम्राज्य के अशोक द्वारा
तीसरी शताब्दी ईसा पव
ू ट के सशलालेखों में है ।

69. दक्षिण भारत में 'सींिम यि


ु ' के बारे में तनम्नसलखखत कथनों पर पवचार करें :

1. उरै यरू कपास व्यापार के सलए प्रससद्ि स्थान था।

2. पह
ु ार कावेरीपट्र्नम के समान है , जो चेर की राजिानी थी।

Target PCS Lucknow Page 42


General Studies Paper#1-Sectional Paper#1

3. पाींड्यों का सबसे प्रारीं सभक सींदभट मेिस्थनीज द्वारा प्रदान ककया िया है ।

उपरोतत में से कौन सा/से कथन सही है /हैं?

A. केवल 1

B. केवल 1 और 3

C. केवल 2 और 3

D. उपरोतत सभी

उत्तर. B

स्पष्र्ीकरण: पह
ु ार दक्षिण भारतीय राज्य तसमलनाडु में मतयलादथ
ु ुराई क्जले का एक शहर है । यह एक
बार एक समद्
ृ ि प्राचीन बींदरिाह शहर था क्जसे कावेरी पम
ू पट्दर्नम के नाम से जाना जाता था, जो
कुछ समय के सलए तसमलकम में प्रारीं सभक चोल राजाओीं की राजिानी के रूप में कायट करता था।
पह
ु ार समि
ु तर् पर कावेरी नदी के मह
ु ाने के पास क्स्थत है ।

70. तनम्नसलखखत कथनों पर पवचार करें :

1. सींिम सादहत्य की रचना पाींड्य राजाओीं के सींरिण में हुई थी।

2. सींिम यि
ु में िन्ने से चीनी बनाई जाती थी।

3. पह
ू ार और र्ोंडी पव
ू ी तर् पर सींिम यि
ु के महत्वपण
ू ट बींदरिाह थे।

ऊपर ददए िए कथनों में से कौन-से सही हैं?

A. केवल 1 और 2

B. केवल 2 और 3

C. केवल 1 और 3

D. 1, 2 और 3

उत्तर. A

Target PCS Lucknow Page 43


General Studies Paper#1-Sectional Paper#1

स्पष्र्ीकरण: सींिम सादहत्य की रचना पाींड्य राजाओीं द्वारा आयोक्जत तीन सींिमों के सींरिण में की
िई थी. सींिम यि
ु का सादहत्य उस काल के जीवन का पवशद वणटन करता है । सींिम कपवताओीं में
रािी और िन्ने की खेती, िन्ने से चीनी बनाने और अनाज सख
ु ाने जैसी कई ग्रामीण िततपवगियों का
वणटन ककया िया है । इस समय के दौरान पह
ु ार पव
ू ी तर् पर एक महत्वपण
ू ट बींदरिाह था और र्ोंडी
पक्चचमी तर् पर एक महत्वपण
ू ट बींदरिाह था।

गुप्ि काल

71. तनम्नसलखखत िप्ु त राजाओीं में से ककसने श्रीलींका के शासक मेघवमाट को िया में बौद्ि मींददर
बनाने की अनम
ु तत दी थी?

A. स्कींदिप्ु त

B. चींििप्ु त प्रथम

C. चींििप्ु त द्पवतीय

D. समि
ु िप्ु त

उत्तर. D

स्पष्र्ीकरण: समि
ु िप्ु त प्राचीन भारत के िप्ु त साम्राज्य के दस
ू रे सम्रार् और भारतीय इततहास के
महानतम शासकों में से एक थे। िप्ु त सम्रार् चींििप्ु त प्रथम और सलच्छवी राजकुमारी कुमारदे वी के
पत्र
ु के रूप में, उन्होंने अपने वींश की राजनीततक शक्तत का बहुत पवस्तार ककया।

72. िप्ु त सोने और चाींदी के ससतके शरू


ु में तनम्नसलखखत में से ककस राज्य के ससतकों पर आिाररत
थे-

A. कुषाण और यौिेयसी

B. कुषाण और शक

C. रोमन और सकास

D. रोमन और कुषाणसी

Target PCS Lucknow Page 44


General Studies Paper#1-Sectional Paper#1

उत्तर. D

स्पष्र्ीकरण: चींििप्ु त प्रथम िप्ु त वींश का एक राजा था, क्जसने उत्तरी भारत में शासन ककया था।
उनकी उपागि महाराजागिराज ("राजाओीं के महान राजा") से पता चलता है कक वह राजवींश के पहले
सम्रार् थे। उनके पत्र
ु समि
ु िप्ु त ने िप्ु त साम्राज्य का और पवस्तार ककया। िप्ु त सोने के ससतकों को
दीनार के रूप में जाना जाता है और वे मि
ु ाशास्त्रीय और कलात्मक उत्कृष्र्ता के सबसे असािारण
उदाहरण हैं। ससतके, सामान्य तौर पर, शासक सम्रार् को अग्रभाि पर गचब्रत्रत करते थे और उसके
दस
ू रे पहलु पर एक दे वी की आकृतत को दशाटया िया था। िप्ु त ससतके शक्ततशाली कुषाणों की नकल
करने लिे, लेककन बहुत जल्द उनकी अपनी पहचान बन िई, जो बाद में आने वाले राजवींशों और
राज्यों के सलए अग्रदत
ू बन िए।

73. प्राचीन भारत में िप्ु त काल के िफ


ु ा गचत्रों के केवल दो ज्ञात उदाहरण हैं। इन्हीीं में से एक है
अजींता की िफ
ु ाओीं की गचत्रकला। िप्ु त गचत्रों का अन्य जीपवत उदाहरण कहााँ है ?

A. एलोरा की िफ
ु ाएीं

B. बाि िफ
ु ाएीं

C. नाससक िफ
ु ाएीं

D. लोमस ऋपष िफ
ु ा

उत्तर. B

स्पष्र्ीकरण: बाि िफ
ु ाएीं मध्य भारत में मध्य प्रदे श राज्य में िार क्जले के बाि शहर में पवींध्य के
दक्षिणी ढलानों के बीच क्स्थत नौ शैलकततटत स्मारकों का एक समह
ू है । ये स्मारक िार शहर से 97
ककमी की दरू ी पर क्स्थत हैं। इन्हें शास्त्रीय िप्ु त काल (319 - 605 ईस्वी) के दौरान बोगिसत्व और
िमटतनरपेि जीवन के गचत्रों से सजाया िया है ।

74. शन्
ू य की अविारणा सदहत दशमलव अींक प्रणाली का आपवष्कार भारत में तनम्नसलखखत में से
ककस राजवींश के दौरान हुआ था?

A. चोल

B. पाल

Target PCS Lucknow Page 45


General Studies Paper#1-Sectional Paper#1

C. िप्ु ता

D. शक

उत्तर. C

स्पष्र्ीकरण: िप्ु त साम्राज्य के दौरान िखणत, खिोल पवज्ञान, पवज्ञान, िमट और दशटन के िेत्र में
भारतीयों की उल्लेखनीय उपलक्ब्ियों के कारण चौथी और छठी शताब्दी आम यि
ु के बीच की अवगि
को भारत के स्वणट यि
ु के रूप में जाना जाता है । इस अवगि के दौरान भारत में शन्
ू य की अविारणा
सदहत दशमलव अींक प्रणाली का आपवष्कार ककया िया था। िप्ु तों के नेतत्ृ व में बनाई िई शाींतत और
समद्
ृ गि ने भारत में वैज्ञातनक और कलात्मक प्रयासों को आिे बढाने में सिम बनाया। भारत में
शन्
ू य की िखणतीय अविारणा लिभि डेढ हजार साल पहले उभरी थी।

75. प्रथम ज्ञात िप्ु त शासक कौन था?

A. कुमारिप्ु त प्रथम

B. घर्ोत्कच

C. चींििप्ु त प्रथम

D. श्रीिप्ु त

उत्तर. D

स्पष्र्ीकरण: श्री िप्ु त िप्ु त वींश का प्रथम शासक था. उन्होंने 240 ईस्वी से 280 ईस्वी तक शासन
ककया। प्रभावती िप्ु त (चींि िप्ु त द्पवतीय की पत्र
ु ी) के पन
ू ा ताींबे के सशलालेख में श्री िप्ु त को िप्ु त
वींश के अगिराज के रूप में वखणटत ककया िया है । उनके शासनकाल के दौरान, उत्तरी या मध्य बींिाल
का दहस्सा िप्ु तों का घर रहा होिा।

76. सच
ू ी I को सच
ू ी II से सम
ु ेसलत करें और नीचे ददए िए कूर् से सही उत्तर का चयन करें :

सच
ू ी I सच
ू ी II

(सशलालेख) (शासक)

a. हाथीिम्
ु फा सशलालेख 1. चोल शासक

Target PCS Lucknow Page 46


General Studies Paper#1-Sectional Paper#1

b. अयोध्या सशलालेख 2. रुिदामण

c. जूनािढ सशलालेख 3. सि
ुीं

d. उत्तरमेरुर सशलालेख 4. खारवेल

कोड:

a b c d

A. 4 3 2 1

B. 3 2 4 1

C. 2 3 4 1

D. 3 4 1 2

उत्तर. A

स्पष्र्ीकरण: हाथीिम्
ु फा सशलालेख खारवेल से सींबगीं ित है . यह उडीसा िेत्र में उसके शासन का वणटन
करता है । अयोध्या सशलालेख शि
ींु वींश से सींबगीं ित है । उत्तरमेरुर सशलालेख, चोल शासकों के समय में ,
ग्राम स्तर पर स्थानीय सरकार के कामकाज का पववरण दे ता है । जूनािढ सशलालेख रुिदामन से
सींबगीं ित है ।

77. असभकथन (A): िप्ु त प्रशासन के दौरान, उपररकस प्राींतों के प्रशासन के प्रभारी अगिकारी थे।

कारण (R): पस्


ु तापाल असभलेख रिक था, क्जसका कतटव्य ककसी भी लेन-दे न को दजट करने से पहले
पछ
ू ताछ करना था।

उपरोतत के सन्दभट में तनम्नसलखखत में से कौन सा सही है

A. A और R दोनों सत्य हैं और R, A की सही व्याख्या करता है ।

B. A और R दोनों सत्य हैं लेककन R, A की सही व्याख्या नहीीं है ।

C. A सच है लेककन R िलत है ।

D. A िलत है लेककन R सत्य है

Target PCS Lucknow Page 47


General Studies Paper#1-Sectional Paper#1

उत्तर. B

स्पष्र्ीकरण: िप्ु त काल के दौरान, परू े साम्राज्य को दे स, या राष्र, या भक्ु तत में पवभाक्जत ककया िया
था। भक्ु ततयों का शासन उपररकों द्वारा ककया जाता था, क्जन्हें सीिे राजा द्वारा तनयत
ु त ककया जाता
था। भक्ु तत प्राींत थे। एक अन्य प्रमख
ु उच्च अगिकारी पस्
ु तपाल (ररकॉडट कीपर) थे। ककसी भी लेनदे न
को ररकॉडट करने से पहले पछ
ू ताछ करना उसका कतटव्य था।

78. असभकथन (A): 'समि


ु िप्ु त' का इलाहाबाद स्तींभ सशलालेख उसी स्तींभ पर उत्कीणट है क्जस पर
शाींततपप्रय अशोक के सशलालेख हैं।

कारण (R): यह सशलालेख उन लोिों और दे शों का उल्लेख करता है क्जन पर समि


ु िप्ु त ने पवजय
प्राप्त की थी।

उपरोतत के सन्दभट में तनम्नसलखखत में से कौन सा सही है

A. A और R दोनों सत्य हैं और R, A की सही व्याख्या करता है ।

B. A और R दोनों सत्य हैं लेककन R, A की सही व्याख्या नहीीं है ।

C. A सच है लेककन R िलत है ।

D. A िलत है लेककन R सत्य है

उत्तर. B

स्पष्र्ीकरण: समि
ु िप्ु त का इलाहाबाद स्तींभ सशलालेख शाही िप्ु तों के राजनीततक इततहास के
अध्ययन के सलए महत्वपण
ू ट असभलेखीय स्रोतों में से एक है । यह उत्कृष्र् सींस्कृत में एक लघक
ु था के
रूप में सलखा िया है । इसे शैिखणक मींडल में इलाहाबाद प्रशक्स्त के नाम से जाना जाता है । यह
उत्कृष्र् सींस्कृत में है , क्जसे कपव और मींत्री, हररषेण द्वारा अगिक पररष्कृत िप्ु त सलपप (ब्राह्मी का
एक बाद का सींस्करण) में सलखा िया है । यह सशलालेख समि
ु िप्ु त की स्ततु त करने वाला एक
गचत्रमय सशलालेख है और दक्षिण में उनके असभयानों सदहत उनके शासनकाल की राजनीततक और
सैन्य उपलक्ब्ियों को सच
ू ीबद्ि करता है ।

79. िप्ु त प्रशासन के बारे में तनम्नसलखखत कथनों पर पवचार करें :

1. मौयट प्रशासन की तल
ु ना में िप्ु त प्रशासन बहुत अगिक पवकेंिीकृत था।

Target PCS Lucknow Page 48


General Studies Paper#1-Sectional Paper#1

2. अगिकाररयों को भसू म अनद


ु ान दे ने की प्रथा मौजूद नहीीं थी।

3. सींगिपवग्रदहका यद्
ु ि और शाींतत मींत्री थे, और बाहरी सींबि
ीं ों से तनपर्ते थे।

ऊपर ददए िए कथनों में से कौन-सा/से सही है /हैं?

A. केवल 1 और 3

B. केवल 2 और 3

C. केवल 3 3

D. 1, 2 और 3

उत्तर. A

स्पष्र्ीकरण: मौयट प्रशासन की तल


ु ना में िप्ु त प्रशासन अगिक पवकेन्िीकृत था। िप्ु त काल के दौरान,
िासमटक मींददरों को भसू म अनद
ु ान दे ने की प्रथा का उदय हुआ। कुछ अगिकाररयों को भसू म अनद
ु ान भी
ददया िया। इस प्रकार, यह भारतीय परीं परा में सामींतवाद की उत्पपत्त थी। सींगिपवग्रदहका यद्
ु ि और
शाींतत मींत्री थे, और बाहरी सींबि
ीं ों से तनपर्ते थे।

80. िप्ु त काल के दौरान कृपष सींरचना के बारे में तनम्नसलखखत कथनों पर पवचार करें :

1. उत्सपाल नामक एक अगिकारी क्जले में सभी भसू म लेनदे न का असभलेख (ररकॉडट) रखता था।

2. वस्ती भसू म का तात्पयट पशच


ु ारण के सलए आरक्षित भसू म से है ।

3. राजा ही भसू म का एकमात्र स्वामी होता था।

ऊपर ददए िए कथनों में से कौन-से सही हैं?

A. केवल 1 और 2

B. केवल 1 और 3

C. केवल 2 और 3

D. 1, 2 और 3

उत्तर. B

Target PCS Lucknow Page 49


General Studies Paper#1-Sectional Paper#1

स्पष्र्ीकरण: पहाडपरु ताम्रपत्र सशलालेख इींगित करता है कक राजा भसू म का एकमात्र मासलक था। यहाीं
तक कक जब उन्होंने भसू म अनद
ु ान ददया, तब भी उन्होंने इस पर अपना पवशेषागिकार सरु क्षित रखा।
जैसा कक पहाडपरु ताम्रपत्र में कहा िया है , उस्तापाल नामक एक अगिकारी ने क्जले में सभी भसू म
लेनदे न का ररकॉडट रखा और ग्राम लेखाकार ने िाींव में भसू म के ररकॉडट को सींरक्षित ककया। िप्ु त काल
के दौरान, भसू म को तनम्नानस
ु ार विीकृत ककया िया था:

(1) िेत्र: कृपष योग्य भसू म

(2) खखला : बींजर भसू म

(3) अपराहत : जींिल या बींजर भसू म

(4) वक्स्त : रहने योग्य भसू म

(5) िपता सराह : पशच


ु ारण भसू म

हर्वर्धवन के बाि हर्वर्धवन और भारि

81. हषटविटन के दरबारी कपव कौन थे?

A. भनीक

B. रपव कीततट

C. बाणभट्र्

D. पवष्णु शमाट

उत्तर. C

स्पष्र्ीकरण: बाणभट्र् राजा हषटविटन के दरबारी कपव थे। सींस्कृत कपव बाणभट्र् द्वारा सलखी िई
उनकी जीवनी हषटचररत ("हषट के कमट"), थानेसर के साथ उनके जुडाव का वणटन करती है , इसके
अलावा रिा दीवार, एक खाई और दो मींक्जला िवलािह
ृ (सफेद हवेली) के साथ महल का उल्लेख
करती है ।

82. राजा हषटविटन द्वारा दो महान िासमटक सम्मेलन आयोक्जत ककए िए थे

Target PCS Lucknow Page 50


General Studies Paper#1-Sectional Paper#1

A. वल्लभी और प्रयाि

B. थानेचवर और वल्लभी

C. प्रयाि और थानेचवरी

D. कन्नौज और प्रयाि

उत्तर. D

स्पष्र्ीकरण: वषट 643 ई. में , हषट ने िींिा नदी के तर् पर कन्नौज में अपनी राजिानी में एक महान
िासमटक सभा की। सभा का उद्दे चय बद्
ु ि की सशिाओीं को उजािर करना था। उस अवसर पर, हषट भी
चीनी कानन
ू के मास्र्र ह्वेन त्साींि का सम्मान करना चाहते थे। कन्नौज पविानसभा के बाद उसी
वषट प्रयाि में एक और शानदार सभा हुई। जबकक कन्नौज पविानसभा महाज्ञानवाद को उजािर करने
के सलए एक िासमटक सभा थी, प्रयाि पविानसभा सभी विों के लोिों को शाही दान दे ने के सलए
सावटभौसमक चररत्र की एक सभा थी। इसे महा मोि पररषद के नाम से जाना जाता था। प्रयाि
पविानसभा में हषट एक उदार सम्रार् और अपने दे श के सभी प्रमख
ु िमों के प्रशींसक के रूप में अपने
सवटश्रेष्ठ प्रदशटन में थे।

83. अपनी जीत के कारण भारत का नेपोसलयन ककसे कहा जाता था?

A. स्कींदिप्ु त

B. चींििप्ु त

C. ब्रह्मिप्ु त:

D. समि
ु िप्ु त

उत्तर. D

स्पष्र्ीकरण: िप्ु त वींश के समि


ु िप्ु त (335-375 ई.) को भारत का नेपोसलयन कहा जाता है ।
इततहासकार ए वी क्स्मथ ने उनका नाम उनकी महान सैन्य पवजय के कारण रखा, जो उनके दरबारी
और कपव हररसेना द्वारा सलखखत 'प्रयाि प्रशक्स्त' से जाना जाता है , जो उन्हें सौ लडाइयों के नायक
के रूप में भी वखणटत करता है ।

Target PCS Lucknow Page 51


General Studies Paper#1-Sectional Paper#1

84. तनम्नसलखखत में से ककस पष्ु यभतू त शासक ने परमभट्र्रक महाराजागिराज की उपागि िारण
की?

A. हषटविटन

B. भट्र्ारक

C. शशाींक

D. प्रभाकरविटन

उत्तर. D

स्पष्र्ीकरण: थानेचवर (ददल्ली के उत्तर में ) पष्ु यभतू त की राजिानी थी. प्रभाकरविटन वींश का सबसे
महत्वपण
ू ट शासक था क्जसने परमभट्र्रक महाराजागिराज की उपागि िारण की थी। मौखररयों के
साथ उनका पववाह सम्बन्ि था। पववाह िठबींिन ने दोनों साम्राज्यों को मजबत
ू ककया। हषटविटन इसी
कुल के थे।

85. तनम्नसलखखत में से ककसकी रचना हषटविटन ने नहीीं की थी?

A. पप्रयदसशटका

B. नािानींद

C. हषटचररत

D. रत्नावली

उत्तर. C

स्पष्र्ीकरण: रत्नावली, नािानींद और पप्रयदसशटका की रचना हषटविटन ने राजनीततक पररक्स्थततयों के


बारे में की थी। उनकी जीवनी हषटचररत ("हषट के कमट") सींस्कृत कपव बाणभट्र् द्वारा सलखी िई थी।

86. सच
ू ी I को सच
ू ी II से सम
ु ेसलत कीक्जए और नीचे ददए िए कूर् से सही उत्तर का चयन कीक्जए:

सच
ू ी I सच
ू ी II

(आगिकाररक) (प्रशासन का िेत्र)

Target PCS Lucknow Page 52


General Studies Paper#1-Sectional Paper#1

a. महाबलागिकृत 1. हाथी ब्रब्रिेड का मखु खया

b. वह
ृ दे चवर 2. घड
ु सवार सेना का मखु खया

c. उपाररक महाराज 3. प्राींतीय मखु खया

d. कर्ुक 4. सेना का सवोच्च अगिकारी

कोड:

a b c d

A. 1 3 2 4

B. 4 2 3 1

C. 2 3 1 4

D. 3 4 1 2

उत्तर. B

स्पष्र्ीकरण: हषटविटन का प्रशासन:-

प्रशासन आगिकाररक िेत्र

महासींिी पवज्जाक अगिकारी यद्


ु ि और शाींतत के बारे में तनणटय लेने वाला अगिकारी

महाबलागिकृत सेना का सवोच्च अगिकारी

बालागिकृत कमाींडर

आयत
ु त सािारण अगिकारी

वह
ृ दे चवर घड
ु सवार सेना के प्रमख

दत
ू राजस्थरुया पवदे श मींत्री

कर्ुक हाथी ब्रब्रिेड के प्रमख


उपाररक महाराज प्राींतीय मखु खया

Target PCS Lucknow Page 53


General Studies Paper#1-Sectional Paper#1

87. असभकथन (A): मिुबन और सोनपत सशलालेख हषट के कालिम को ररकॉडट करते हैं।

कारण (R): बींशख


े रा असभलेखों में हषट के हस्तािर हैं।

उपरोतत के सन्दभट में तनम्नसलखखत में से कौन सा सही है

A. A और R दोनों सत्य हैं और R, A की सही व्याख्या करता है ।

B. A और R दोनों सत्य हैं लेककन R, A की सही व्याख्या नहीीं है ।

C. A सच है लेककन R िलत है ।

D. A िलत है लेककन R सत्य है

उत्तर. B

स्पष्र्ीकरण: ह्वेन त्साींि के यात्रा वत्त


ृ ाींत सी-य-ू की, बनभट्र् के हषटचररत और पल
ु केसशन द्पवतीय के
ऐहोल पत्थर के सशलालेख ने पष्ु यभतू त राजवींश के बारे में बताया। मिुबन और सोनपत असभलेखों में
हषट का कालिम दजट है । बींशख
े रा असभलेखों में हषट के हस्तािर हैं। पष्ु यभतू त िप्ु त के सामींत थे। हूणों
के आिमण के बाद वे स्वतींत्र हुए। इस राजवींश के महत्वपण
ू ट शासक प्रभाकरविटन, आददत्यविटन और
हषटविटन थे। हषटविटन प्रभाकरविटन के पत्र
ु थे।

88. असभकथन (A): अपभ्रींश कपव, स्वयींभू प्रततहारों के दरबार में रहते थे।

कारण (R): सींस्कृत कपव और नार्ककार राजशेखर मदहपाल के दरबार में रहते थे।

उपरोतत के सन्दभट में तनम्नसलखखत में से कौन सा सही है

A. A और R दोनों सत्य हैं और R, A की सही व्याख्या करता है ।

B. A और R दोनों सत्य हैं लेककन R, A की सही व्याख्या नहीीं है ।

C. A सच है लेककन R िलत है ।

D. A िलत है लेककन R सत्य है

उत्तर. D

स्पष्र्ीकरण: राजशेखर, भोज के पोते मदहपाल के दरबार में रहते थे। महान अपभ्रींश कपव, स्वयींभू
और उनके पत्र
ु , राष्रकूर्ों के दरबार में रहते थे।

Target PCS Lucknow Page 54


General Studies Paper#1-Sectional Paper#1

89. तनम्नसलखखत कथनों को ध्यानपव


ू क
ट पदढए और नीचे ददए िए कूर् का प्रयोि कर सही उत्तर
चुतनए:

1. हषटचररत बाण भट्र् द्वारा सलखा िया था।

2. हषट के दक्षिण की ओर माचट को पल


ु केसशन द्पवतीय चालत
ु य द्वारा ताप्ती नदी के तर् पर रोक
ददया िया था।

3. सी-यू-की के अनस
ु ार हषट के राजस्व को चार भािों में बाींर्ा िया था।

ऊपर ददए िए कथनों में से कौन-सा/से सही है /हैं?

A. केवल 1 और 2

B. केवल 1 और 3

C. केवल 2 और 3

D. उपरोतत सभी

उत्तर. B

स्पष्र्ीकरण: हषटविटन के शासन के दौरान, कन्नौज को जीत सलया िया और थानेचवर के साथ समला
ददया िया। हषटविटन ने िौडा साम्राज्य के शशाींक पर हमला ककया और बींिाल, ब्रबहार और ओडडशा के
िेत्रों पर अपना तनयींत्रण स्थापपत ककया और कामरूप (असम) के भास्करवमटन से समत्रता की। िज
ु रात
में वल्लभी राजा ध्रुवभट्र् भी हार िए थे और हषट की बेर्ी का ध्रुवभट्र् से पववाह करके उनके साथ
एक समझौता ककया िया था। उन्होंने उत्तरापथनाथ या उत्तरपथपतत (उत्तर के भिवान) की उपागियााँ
ग्रहण कीीं। दक्षिण की ओर बढते हुए, हषटविटन ने मलावा के िेत्रों पर पवजय प्राप्त की और नमटदा को
पार करने के बाद वह नमटदा के यद्
ु ि में पल
ु केसशन द्पवतीय से हार िया।

90. हषटविटन के बारे में तनम्नसलखखत कथनों पर पवचार करें :

1. उसके शासनकाल में कानन


ू -व्यवस्था ठीक नहीीं थी।

2. दक्ण्डन ने हषटचररत की रचना की।

3. हषट महायान बौद्ि िमट के सींरिक थे।

Target PCS Lucknow Page 55


General Studies Paper#1-Sectional Paper#1

ऊपर ददए िए कथनों में से कौन-से सही हैं?

A. केवल 1 और 2

B. केवल 2 और 3

C. केवल 1 और 3

D. 1, 2 और 3

उत्तर. C

स्पष्र्ीकरण: हषट ने एक सदहष्णु िासमटक नीतत का पालन ककया. अपने प्रारीं सभक वषों में एक शैव, वह
िीरे -िीरे बौद्ि िमट का एक महान सींरिक बन िया। एक िमटतनष्ठ बौद्ि के रूप में , उन्होंने
महायान के ससद्िाींतों को व्यापक रूप से प्रचाररत करने के सलए कन्नौज में एक भव्य सभा बल
ु ाई।
बाणभट्र् हमें अपनी पस्
ु तक हषटचररत में अपने सींरिक के प्रारीं सभक वषों का एक पववरण दे ता है । हषट
को न केवल उनके सींरिण और अगििम (learning) के सलए याद ककया जाता है , बक्ल्क तीन नार्कों
पप्रयदसशटका, रत्नावली और नािानींद के लेखन के सलए भी याद ककया जाता है ।

प्रारिं सभक र्ध्यकालीन भारि

91. प्रारीं सभक मध्यकालीन भारत के तनम्नसलखखत शासक वींश में से कौन खुद को सात्यकक (Satyaki)
का वींशज कहता था?

A. पाल

B. राष्रकूर्

C. चोल

D. पवजयनिर

उत्तर. B

स्पष्र्ीकरण: राष्रकूर् छठी और दसवीीं शताब्दी के बीच भारतीय उपमहाद्वीप के बडे दहस्से पर शासन
करने वाला एक शाही राजवींश था। सबसे पहले ज्ञात राष्रकूर् सशलालेख, मध्य या पक्चचम भारत के
एक शहर, मनापरु ा से उनके शासन का पववरण दे ने वाला सातवीीं शताब्दी का ताम्रपत्र अनद
ु ान है ।

Target PCS Lucknow Page 56


General Studies Paper#1-Sectional Paper#1

राष्रकूर् स्वयीं को सात्यकक का वींशज कहते थे। राष्रकूर् शक्तत के सींस्थापक दीं तीवमाट या दीं ततदि
ु ट थे
जो चालत
ु य राजा पल
ु केसशन द्पवतीय के समकालीन थे।

92. तनम्नसलखखत में से ककस राजवींश ने खजुराहो के मींददरों का तनमाटण करवाया था?

A. चींदेल

B. चाहमान

C. परमार

D. तोमर

उत्तर. A

स्पष्र्ीकरण: खजुराहो समह


ू के मींददरों का तनमाटण राजपत
ू चींदेल वींश के शासन के दौरान हुआ था.
उन्होंने अपने परू े राज्य में सत्ता में आते ही पररसर का तनमाटण शरू
ु कर ददया, क्जसे बाद में बद
ुीं े लखींड
के नाम से जाना जाने लिा। दहींद ू राजाओीं यशोवमटन और िींिा के शासनकाल के दौरान अगिकाींश
मींददरों का तनमाटण 950 और 1050 ईस्वी के बीच ककया िया था।

93. तनम्नसलखखत में से ककस प्रारीं सभक मध्ययि


ु ीन भारतीय राजवींश ने एलोरा में प्रससद्ि कैलाश
(सशव) का शैलकततटत मींददर बनवाया था?

A. चालत
ु य राजवींश

B. चोल राजवींश

C. राष्रकूर् राजवींश

D. पाल राजवींश

उत्तर. C

स्पष्र्ीकरण: एलोरा में कैलाश (सशव) का प्रससद्ि शैलकततटत मींददर राष्रकूर् राजा कृष्ण प्रथम द्वारा
बनाया िया था। कृष्ण प्रथम ने 757 में अींततम बादामी चालत
ु य शासक कीततटवमटन द्पवतीय को
हराकर बढते राष्रकूर् साम्राज्य का प्रभार सींभाला था। यह 807 के सम्रार् िोपवींदा III का ताम्रपत्र
अनद
ु ान और बडौदा से िज
ु रात राष्रकूर् सम्रार् करका का ताम्रपत्र अनद
ु ान से जाना जाता है ।

Target PCS Lucknow Page 57


General Studies Paper#1-Sectional Paper#1

94. तनम्नसलखखत में से कौन सा यात्री पाल साम्राज्य को रूहमा के रूप में सींदसभटत करता है ?

A. ब्रत्रब्रबिम

B. सल
ु ेमान

C. सभ
ु ाति
ींु ा

D. अकालवषट

उत्तर. B

स्पष्र्ीकरण: सल
ु ेमान या सल
ु ेमान अल-ताजीर 9वीीं शताब्दी के फारसी मक्ु स्लम व्यापारी, यात्री और
लेखक थे। वह पाल साम्राज्य को 'रुहमा' कहता है और उनकी सैन्य शक्तत को प्रमाखणत करता है ।

95. तनम्नसलखखत में से कौन कचमीर का शासक विट नहीीं है ?

A. काकोर्

B. उत्पल

C. लोहार

D. सेन

उत्तर. D

स्पष्र्ीकरण: कचमीर पर तीन राजवींशों का शासन था- काकोर्, उत्पल और लोहार। लसलताददत्य
प्रारीं सभक मध्ययि
ु ीन भारत के दौरान कचमीर का सबसे प्रससद्ि शासक था। बींिाल पर सेन वींश का
शासन था। भारतीय उपमहाद्वीप पर प्रारीं सभक मध्ययि
ु ीन काल के दौरान सेन साम्राज्य एक दहींद ू
राजवींश था, क्जसने 11वीीं और 12वीीं शताब्दी तक बींिाल पर शासन ककया था।

96. सच
ू ी I को सच
ू ी II से सम
ु ेसलत कीक्जए और नीचे ददए िए कूर् से सही उत्तर का चयन कीक्जए:

सच
ू ी I सच
ू ी II

Target PCS Lucknow Page 58


General Studies Paper#1-Sectional Paper#1

(मध्यकालीन भारत के शब्द) (अथट)

a. खराज 1. पविोही िााँव जहााँ बल प्रयोि से भ-ू राजस्व की वसल


ू ी की जाती
थी

b. पोलज 2. वापषटक खेती योग्य भसू म

c. खालसा 3. भ-ू राजस्व

d. मावा 4. शाही राजस्व पवभाि द्वारा सीिे प्रशाससत राजस्व दे ने वाली


भसू म

कोड:

a b c d

A. 4 3 2 1

B. 3 2 4 1

C. 2 3 4 1

D. 3 4 1 2

उत्तर. B

स्पष्र्ीकरण: खराज एक भसू म कर था जो मल


ू रूप से केवल िैर-मस
ु लमानों द्वारा भि
ु तान ककया
जाता था। बाद में , खराज शब्द मक्ु स्लम अगिकाररयों को भि
ु तान ककए िए ककसी भी कर का वणटन
करने के सलए आता है । वतफ जैसे कुछ सींस्थानों को दोनों से छूर् दी िई थी। पोलज वह भसू म है
क्जस पर ब्रबना ककसी रुकावर् के सालाना खेती की जाती थी। मावा एक ऐसा इलाका था क्जसमें
पविोही मखु खया और ककसान रहते थे जो ब्रबना लडाई के सल्
ु तान के अगिकाररयों को भ-ू राजस्व का
भि
ु तान नहीीं करते थे। अतसर मावा िेत्र ददल्ली सल्तनत के पविोही रईसों की शरणस्थली के रूप में
कायट करते थे। शाही राजस्व पवभाि द्वारा सीिे प्रशाससत राजस्व दे ने वाली भसू म को खालसा के नाम
से जाना जाता था। आमतौर पर सबसे उपजाऊ और आसानी से प्रशाससत भसू म खालसा के भीतर लाई
जाती थी।

97. असभकथन (A): मख्


ु य रूप से, बहमनी साम्राज्य की राजिानी िल
ु बिाट थी।

Target PCS Lucknow Page 59


General Studies Paper#1-Sectional Paper#1

कारण (R): अहमद शाह वली ने राजिानी को िल


ु बिाट से बीदर (bidar) स्थानाींतररत कर ददया।

उपरोतत के सन्दभट में तनम्नसलखखत में से कौन सा सही है

A. A और R दोनों सत्य हैं और R, A की सही व्याख्या करता है ।

B. A और R दोनों सत्य हैं लेककन R, A की सही व्याख्या नहीीं है ।

C. A सच है लेककन R िलत है ।

D. A िलत है लेककन R सत्य है

उत्तर. B

स्पष्र्ीकरण: बहमनी सल्तनत दक्षिण भारत में दतकन (deccan) का एक फारसी मक्ु स्लम साम्राज्य
था। यह दतकन का पहला स्वतींत्र मक्ु स्लम राज्य था, और पवजयनिर के अपने दहींद ू प्रततद्वींद्पवयों के
साथ अपने सतत यद्
ु िों के सलए जाना जाता था। बहमनी साम्राज्य की स्थापना 1347 में अलाउद्दीन
हसन ने की थी। अपने राज्यासभषेक के बाद, उन्होंने अलाउद्दीन हसन बहमन शाह (1347-58) की
उपागि िारण की, इस उपागि से ही राज्य को बहमनी साम्राज्य कहा िया। 1347 और 1425 के
बीच बहमनी राजिानी अहसानाबाद (िल
ु बिाट) थी, जब इसे मह
ु म्मदाबाद (बीदर) में स्थानाींतररत कर
ददया िया था।

98. असभकथन (A): तक


ु ों ने एक बेहतर चूना (mortar) पेश ककया क्जसने उन्हें मेहराब और िब
ुीं द के
आिार पर शानदार इमारतों को खडा करने में सिम बनाया।

कारण (R): तक
ु ों ने 'राहत' में सि
ु ार ककया क्जससे ससींचाई के सलए िहरे कुओीं से पानी तनकालने का
काम आसान हो िया।

उपरोतत के सन्दभट में तनम्नसलखखत में से कौन सा सही है

A. A और R दोनों सत्य हैं और R, A की सही व्याख्या करता है ।

B. A और R दोनों सत्य हैं लेककन R, A की सही व्याख्या नहीीं है ।

C. A सच है लेककन R िलत है ।

D. A िलत है लेककन R सत्य है

उत्तर. B

Target PCS Lucknow Page 60


General Studies Paper#1-Sectional Paper#1

स्पष्र्ीकरण: परु ाने समय में ससींचाई की राहत प्रणाली का उपयोि बैलों का उपयोि करके कुएीं से
पानी प्राप्त करने के तरीके के रूप में ककया जाता था जो मख्
ु य रूप से उपमहाद्वीप में उपयोि ककया
जाता था। कुएीं से पानी तनकालने के सलए बैलों, भैंसों या िायों द्वारा पदहया घम
ु ाया जाता है और
कफर फसलों पर फैला ददया जाता है । एक बेहतर मोर्ाटर की शरू
ु आत ने तक
ु ों को मेहराब और िब
ुीं द के
आिार पर शानदार इमारतों को खडा करने में सिम बनाया।

99. तनम्नसलखखत में से कौन सा/से कथन सही है /हैं?

1. मध्यकालीन भारत की अवगि के दौरान, मखणग्रामम और नानादे शी सशिा केंि थे।

2. होयसालेचवर मींददर चालत


ु य शैली का एक उदाहरण है ।

3. नयनार पवष्णु के भतत थे।

नीचे ददए िए कूर् का प्रयोि कर सही उत्तर चुतनए:

A. 2 केवल

B. केवल 2 और 3

C. 1 और 2 केवल

D. 1, 2 और 3

उत्तर. A

स्पष्र्ीकरण: मध्यकालीन भारत की अवगि के दौरान, भारतीय व्यापाररयों को सींघों में सींिदठत ककया
िया था, उनमें से सबसे प्रससद्ि मखणग्रामम और नानादे शी थे, जो प्रारीं सभक काल से सकिय थे।
चोलों के पतन के बाद, कल्याणी के चालत
ु यों और होयसलों के अिीन मींददर तनमाटण कायट जारी रहा।
िारवाड क्जले और होयसल की राजिानी हलेब्रबड में बडी सींख्या में मींददर थे। इनमें से सबसे भव्य
होयसलेचवर मींददर है । यह चालत
ु य शैली का सवोत्तम उदाहरण है । कई लोकपप्रय सींत, क्जन्हें नयनार
और आलवार कहा जाता है , जो िमशः सशव और पवष्णु के भतत थे, 6वीीं और 9वीीं शताब्दी के बीच
तसमल राज्यों में फले-फूले।

100. तनम्नसलखखत में से कौन से कथन सही हैं?

Target PCS Lucknow Page 61


General Studies Paper#1-Sectional Paper#1

1. चालत
ु य राजा पल
ु केसशन द्पवतीय ने हषट को पराक्जत ककया था और उसे दतकन की ओर अपने
राज्य का पवस्तार करने की अनम
ु तत नहीीं दी थी।

2. चोल साम्राज्य का सींस्थापक पवजयालय था।

3. राजराजा और राजेंि प्रथम ने पवसभन्न स्थानों पर कई सशव और पवष्णु मींददरों का तनमाटण करके
अपनी जीत को गचक्ह्नत ककया।

4. राजराजा ने िींिईकोंडाचोला (Gangaikondachola) की उपागि िारण की।

नीचे ददए िए कूर् का प्रयोि कर सही उत्तर चुतनए:

A. केवल 1, 2 और 3

B. केवल 2 और 4

C. केवल 1 और 3

D. 1, 2, 3 और 4

उत्तर. A

स्पष्र्ीकरण: चोल साम्राज्य का सींस्थापक पवजयालय था, जो पहले पल्लवों का सामींत था। उसने 850
ईस्वी में तींजौर पर कब्जा कर सलया। 9वीीं शताब्दी के अींत तक, चोलों ने काींची (तोंडईमींडलम) के
दोनों पल्लवों को हरा ददया और पाींड्यों को कमजोर कर ददया, क्जससे दक्षिणी तसमल दे श अपने
तनयींत्रण में आ िया। राजराजा और राजेंि प्रथम ने पवसभन्न स्थानों पर कई सशव और पवष्णु मींददरों
का तनमाटण करके अपनी जीत को गचक्ह्नत ककया। इनमें से सबसे प्रससद्ि तींजौर में बह
ृ दीचवर मींददर
था, जो 1010 में परू ा हुआ था। राजेंि प्रथम के शासनकाल में सबसे उल्लेखनीय कारनामों में से एक
कसलींि से बींिाल तक माचट था, क्जसमें चोल सेनाओीं ने िींिा नदी को पार ककया और दो स्थानीय
राजा को हराया। यह असभयान, क्जसका नेतत्ृ व एक चोल सेनापतत ने ककया था, 1022 में हुआ और
उसी मािट का उल्र्ा चल रहा था, क्जसका महान पवजेता समि
ु िप्ु त ने अनस
ु रण ककया था। इस
अवसर को मनाने के सलए, राजेंि प्रथम ने िींिईकोंडाचोला (िींिा पर पवजय प्राप्त करने वाले चोल)
की उपागि िारण की। उन्होंने कावेरी नदी के मह
ु ाने के पास एक नई राजिानी का तनमाटण ककया और
इसे िींिईकोंडाचोलपरु म (िींिा पर पवजय प्राप्त करने वाले चोल का शहर) कहा।

दिल्ली िल्िनि

Target PCS Lucknow Page 62


General Studies Paper#1-Sectional Paper#1

101. तनम्नसलखखत में से ककसने ददल्ली के सात सल्


ु तानों का शासन दे खा?

A. अमीर खुसरो

B. इनमें से कोई नहीीं

C. शम्स ससराज अफीफी

D. ख्वाजा मोइनद्
ु दीन गचचती

उत्तर. A

स्पष्र्ीकरण: अमीर खुसरो एक सफ


ू ी फकीर और तनजामद्
ु दीन औसलया के आध्याक्त्मक सशष्य थे। वह
72 वषों तक जीपवत रहा, क्जसमें से 60 वषट वह ददल्ली सल्तनत के दस पवसभन्न शासकों के दरबार
में रहा। वह एक कपव होने के साथ-साथ पवपल
ु सींिीतकार भी थे। कपवताएाँ सलखने के सलए उनकी
प्राथसमक भाषा फारसी थी लेककन उन्होंने फारसी, तक
ु ी, अरबी, ब्रज भाषा, दहींदवी और साथ ही खडी
बोली में लिभि पाींच लाख छीं दों की रचना की। उनकी खासलक-ए-बारी, जो दतु नया के सबसे परु ाने
मदु ित शब्दकोश के रूप में जानी जाती है , दहींदी और फारसी शब्दों से सींबगीं ित है । उन्हें "कव्वाली के
पपता" के रूप में माना जाता है , उन्हें लिातार तीन खखलजी शासकों द्वारा सींरिण ददया िया था।

102. तनम्नसलखखत में से कौन मह


ु म्मद िोरी का दास नहीीं था?

A. कुतब
ु द्
ु दीन ऐबक

B. शमसद्
ु दीन इल्तत
ु समश

C. ताजुद्दीन यलदौजी

D. इक्ख्तयारुद्दीन बक्ख्तयार खखलजी

उत्तर. D

स्पष्र्ीकरण: इक्ख्तयारुद्दीन बक्ख्तयार खखलजी कुतब


ु द्
ु दीन ऐबक का एक तक
ु ी सैन्य जनरल था। वह
मह
ु म्मद िोरी का िल
ु ाम नहीीं था।

103. ददल्ली सल्तनत काल में भसू म की माप के सलए प्रयत


ु त शब्द था

Target PCS Lucknow Page 63


General Studies Paper#1-Sectional Paper#1

A. घल्ला बख्शी

B. घोरी

C. मसाहत

D. ककस्मत-ए-घल्ला

उत्तर. C

स्पष्र्ीकरण: ददल्ली सल्तनत काल में भसू म की माप के सलए मसाहत शब्द का प्रयोि ककया जाता
था। अलाउद्दीन खखलजी और मोहम्मद ब्रबन ति
ु लक जैसे शासकों ने साम्राज्य के राजस्व को
अगिकतम और सव्ु यवक्स्थत करने के िम में माप द्वारा भ-ू राजस्व के आकलन की एक प्रणाली
बनाने के सलए इसका उपयोि ककया।

104. ददल्ली के सल्


ु तानों के अिीन तनम्नसलखखत में से कौन सी एक मि
ु ा प्रचलन में नहीीं थी?

A. जीतल

B. डोिनी

C. र्ीं का

D. दाम

उत्तर. D

स्पष्र्ीकरण: दाम ददल्ली के सल्


ु तानों के अिीन प्रचलन में आने वाली मि
ु ा नहीीं थी। यह शेर शाह के
शासनकाल के दौरान पररचासलत ककया िया था और ताींबे से बना था, जो एक रुपये के ससतके के
1/40वें दहस्से के बराबर था। इल्तत
ु समश ने दो ससतके पेश ककए जो ददल्ली सल्तनत के बाद के
ससतके का आिार बने चाींदी का र्ीं का और ताींबे का जीतल।

105. तनम्नसलखखत में से ककसने तक


ु ाटन-ए-चहलिानी या चालीस का आयोजन ककया?

A. कुतब
ु द्
ु दीन ऐबक

B. रक्जया सल्
ु तान

Target PCS Lucknow Page 64


General Studies Paper#1-Sectional Paper#1

C. नसीरुद्दीन महमद

D. इल्तत
ु समश

उत्तर. D

स्पष्र्ीकरण: इल्तत
ु समश जो ददल्ली के िल
ु ाम वींश के तीसरे शासक थे, ने 40 वफादार िल
ु ाम अमीरों
का एक दल िदठत ककया, क्जन्हें तक
ु ाटन-ए-चहलिानी के नाम से जाना जाता है । यह तक
ु ी कुलीन विट
का चयतनत तनकाय था क्जसे बाद में बलबन ने नष्र् कर ददया था। ये तक
ु ी रईस सल्तनत के
प्रशासन में सल्
ु तान को सलाह और मदद करते थे।

106. तनम्नसलखखत में से कौन ददल्ली के ससींहासन पर बैठने वाला एकमात्र दहींद ू िमांतररत था?

A. अलाउद्दीन खखलजी

B. जलाउद्दीन खखलजी

C. मब
ु ारक शाह

D. खस
ु रो शाह

उत्तर. D

स्पष्र्ीकरण: खुसरो खान 1320 में लिभि दो महीनों के सलए ददल्ली के सल्
ु तान थे। वह बाराडू दहींद ू
सैन्य कबीले से सींबगीं ित थे, और 1305 में अलाउद्दीन खखलजी की मालवा पर पवजय के दौरान
ददल्ली की सेना ने उन्हें पकड सलया था। नसीरुद्दीन खुसरो शाह ददल्ली सल्तनत के प्रमख
ु बनने
वाले एकमात्र पररवततटत भारतीय मस
ु लमान थे। वह अलाउद्दीन खखलजी की सेना में एक सेनापतत था
और उसने अलाउद्दीन के उत्तरागिकारी मब
ु ारक शाह खखलजी की हत्या कर दी और बहुत कम समय
के सलए ददल्ली का सल्
ु तान बना।

107. मक्ु न्हयान तनम्नसलखखत में से ककस अगिकारी को सींदसभटत करता है ?

A. खुकफया अगिकारी

B. राजस्व अगिकारी

C. िप्ु त जासस

Target PCS Lucknow Page 65


General Studies Paper#1-Sectional Paper#1

D. उपरोतत में से कोई नहीीं

उत्तर. C

स्पष्र्ीकरण: मक्ु न्हयान िप्ु त जासस


ू थे क्जन्हें अलाउद्दीन खखलजी ने तनयत
ु त ककया था। अलाउद्दीन
को बरीदों से जो खुकफया अगिकारी थे और मक्ु न्हयानों से बाजार की दै तनक ररपोर्ट प्राप्त होती थी।
शाहना-ए-मींडी नामक बाजार के अिीिक को एक खकु फया अगिकारी द्वारा सहायता प्रदान की जाती
थी।

108. सल्तनत काल में 'वकील-ए-दार' का तया कायट था?

A. अदालती मयाटदा बनाए रखना

B. कानन
ू और व्यवस्था बनाए रखना

C. िासमटक मामलों का ध्यान रखना

D. उपरोतत में से कोई नहीीं

उत्तर. A

स्पष्र्ीकरण: सल्तनत के अिीन वकील या वकील-ए-दार का कायाटलय शाही घराने के प्रबींिन से


सींबगीं ित था। सल्तनत काल के दौरान वकील-ए-दार के रूप में जाना जाने वाला अगिकारी अदालत की
मयाटदा बनाए रखने के सलए क्जम्मेदार था। उन्होंने दरबार में रईसों के बैठने की व्यवस्था का भी
तनरीिण करता था।

109. 'अलाई दरवाजा' और ससरी ककले का तनमाटण ककसने करवाया था?

A. ियासद्
ु दीन ति
ु लक

B. कफरोज शाह ति
ु लक

C. अलाउद्दीन खखलजी

D. मो. ब्रबन ति
ु लक

उत्तर. C

Target PCS Lucknow Page 66


General Studies Paper#1-Sectional Paper#1

स्पष्र्ीकरण: अलाउद्दीन खखलजी ने अपने शासनकाल में कला और वास्तक


ु ला का सींरिण ककया
तयोंकक उन्होंने अलाई दरवाजा (कुतब
ु मीनार का प्रवेश द्वार), ससरी ककला, हजार ससतरु , हौज-ए-
अलाई, आदद जैसे कई ककलों का तनमाटण ककया था। अलाई दरवाजा कुतब
ु पररसर, महरौली, ददल्ली,
भारत में कुव्वत-उल-इस्लाम मक्स्जद का दक्षिणी प्रवेश द्वार है । 1311 में सल्
ु तान अलाउद्दीन
खखलजी द्वारा तनसमटत और लाल बलआ
ु पत्थर से बना, यह एक चौकोर िब
ुीं द वाला िेर्हाउस
(चौकीदार का घर) है क्जसमें िनष
ु ाकार प्रवेश द्वार हैं और इसमें एक ही कि है । उन्होंने भारत और
ददल्ली के मींिोल आिमणों से बचाव के सलए 1297 और 1307 के बीच ससरी का तनमाटण ककया।
जवाब में , उसने ससरी ककले का तनमाटण ककया।

110. चाींदी के र्ीं का और जीतल दो प्रमख


ु ससतके ककसने पेश ककए?

A. ियासद्
ु दीन बलबन

B. कुतब
ु द्
ु दीन ऐबक

C. शम्सद्
ु दीन इल्तत
ु समश

D. अलाउद्दीन खखलजी

उत्तर. C

स्पष्र्ीकरण: र्ीं का एक चााँदी का ससतका था। र्ीं का 48 जीतल के बराबर था। शशिनी एक चाींदी का
ससतका था जो 6 जीतल के बराबर होता था। जीतल ताींबे का ससतका था। इल्तत
ु समश ने ददल्ली
सल्तनत के दो ससतकों चाींदी का र्ीं का और ताम्बे का जीतल को पेश ककया। इल्तत
ु समश भारत में
"शद्
ु ि अरबी ससतका" पेश करने वाले पहले व्यक्तत थे। बिदाद के खलीफा से ददल्ली के सींप्रभु
सल्
ु तान का एक अलींकरण प्राप्त करने के बाद ससतकों को "माइर्ी सल्
ु तान, एम्पायर ऑफ द एम्पायर
एींड द फेथ, कानतवेस्र्-लादे न, इल-तत
ु समश" के साथ उकेरा िया था। इल्तत
ु समश द्वारा जारी चाींदी
का र्ीं का 175 ग्रेन का था। बाद में बलबन ने उसी वजन का सोने का र्ीं का जारी ककया था।

111. शेर शाह के प्राींतीय प्रशासन के सींदभट में , "सशकदार" तनम्नसलखखत में से ककसके सलए क्जम्मेदार
था?

A. कानन
ू और व्यवस्था, और सामान्य प्रशासन

B. भ-ू राजस्व का सींग्रहण

Target PCS Lucknow Page 67


General Studies Paper#1-Sectional Paper#1

C. िासमटक सींस्कार

D. न्याय के सलए क्जम्मेदार

उत्तर. A

स्पष्र्ीकरण: कई िाींवों में एक परिना शासमल था। परिना सशकदार के अिीन था, जो कानन
ू और
व्यवस्था और सामान्य प्रशासन की दे खभाल करता था। मसुीं सफ या आसमल भ-ू राजस्व के सींग्रह की
दे खभाल करते थे। खाते फारसी और स्थानीय भाषा (दहींदवी) दोनों में बनाए िए थे। परिने के ऊपर
सशकदार-ए-सशकदारन या फौजदार और एक मसींु सफ-ए-मन्
ु सीफान के अिीन सशक या सरकार थी। ऐसा
लिता है कक इन अगिकाररयों के केवल पद ही नए थे, तयोंकक परिना और सरकार दोनों ही पहले के
काल में भी प्रशासन की इकाइयााँ थीीं।

112. ककस ददल्ली सल्


ु तान के शासनकाल के दौरान सींस्कृत ग्रींथों का अरबी-फारसी और इसके
पवपरीत अनव
ु ाद करने के सलए अनव
ु ाद पवभाि की स्थापना की िई थी?

A. मोहम्मद ति
ु लक

B. रक्जया सल्
ु तान

C. कुतब
ु द्
ु दीन मब
ु ारक शाह खखलजी

D. ससकींदर लोदी

उत्तर. D

स्पष्र्ीकरण: ससकींदर लोदी के शासनकाल के दौरान, सींस्कृत ग्रींथों का अरबी-फारसी और इसके


पवपरीत अनव
ु ाद करने के सलए राज्य अनव
ु ाद पवभाि की स्थापना की िई थी। वे पवद्या के सींरिक
थे।

113. सच
ू ी I को सच
ू ी II से सम
ु ेसलत कीक्जए और नीचे ददए िए कूर् से सही उत्तर का चयन कीक्जए:

सच
ू ी I सच
ू ी II

(पवभाि) (कायट की प्रकृतत)

a. दीवान-ए-एररज 1. कृपष पवभाि

Target PCS Lucknow Page 68


General Studies Paper#1-Sectional Paper#1

b. दीवान-ए-मस्
ु तखराज 2. बकाया पवभाि

c. दीवान-ए-कोही 3. सैन्य पवभाि

d. दीवान-ए-बींदिान 4. िल
ु ामों का पवभाि

कोड:

a b c d

A. 4 3 2 1

B. 3 2 1 4

C. 2 3 4 1

D. 3 4 1 2

उत्तर. B

स्पष्र्ीकरण:

S.N. वर्भाग कायव की प्रकृति


1. दीवान-ए-एररज सैन्य पवभाि
2. दीवान-ए-मस्
ु तखराज बकाया पवभाि
3. दीवान-ए-कोही कृपष पवभाि
4. दीवान-ए-बींदिान दासों का पवभाि

114. असभकथन (A): कुतब


ु द्
ु दीन ऐबक ने दो मक्स्जदों का तनमाटण ककया, कुवानल
ु - ददल्ली में
इस्लाम मक्स्जद और आमेर में ढाई ददन का झोंपडा।

कारण (R): कुतब


ु द्
ु दीन ऐबक ने एक सफ
ू ी सींत ख्वाजा कुतब
ु द्
ु दीन बक्ख्तयार काकी के नाम पर कुतब

मीनार की नीींव रखी।

उपरोतत के सन्दभट में तनम्नसलखखत में से कौन सा सही है

A. A और R दोनों सत्य हैं और R, A की सही व्याख्या करता है ।

B. A और R दोनों सत्य हैं लेककन R, A की सही व्याख्या नहीीं है ।

Target PCS Lucknow Page 69


General Studies Paper#1-Sectional Paper#1

C. A सच है लेककन R िलत है ।

D. A िलत है लेककन R सत्य है

उत्तर. B

स्पष्र्ीकरण: मध्यकालीन भारत के शासक कुतब


ु द्
ु दीन ऐबक ददल्ली सल्तनत के पहले शासक और
िल
ु ाम वींश के सींस्थापक भी थे। वह अयबक जनजातत का एक तक
ु ी था और केवल चार साल,
1206-1210 के सलए सल्
ु तान था। ढाई ददन का झोंपडा वास्तव में 1199 ई. में ददल्ली के पहले
सल्
ु तान कुतब
ु -उद-दीन-ऐबक द्वारा तनसमटत एक मक्स्जद है । यह ददल्ली के कुतब
ु -मीनार पररसर में
बनी दस
ू री मक्स्जद के समकालीन है क्जसे कुवाल-उल-इस्लाम (इस्लाम की शक्तत) मक्स्जद के नाम
से जाना जाता है ।

115. असभकथन (A): अहमद शाह प्रथम को िज


ु रात राज्य का वास्तपवक सींस्थापक माना जाता है ।

कारण (R): उन्होंने िज


ु रात के जैतनयों की समद्
ृ ि स्थापत्य परीं पराओीं का उपयोि इमारत की एक
शैली तैयार करने के सलए ककया जो ददल्ली से स्पष्र् रूप से अलि थी।

उपरोतत के सन्दभट में तनम्नसलखखत में से कौन सा सही है

A. A और R दोनों सत्य हैं और R, A की सही व्याख्या करता है ।

B. A और R दोनों सत्य हैं लेककन R, A की सही व्याख्या नहीीं है ।

C. A सच है लेककन R िलत है ।

D. A िलत है लेककन R सत्य है

उत्तर. B

स्पष्र्ीकरण: िज
ु रात राज्य के वास्तपवक सींस्थापक मज
ु फ्फर शाह के पोते अहमद शाह प्रथम (1411–
43) थे। अपने लींबे शासनकाल के दौरान, वे कुलीनता को तनयींत्रण में लाया, प्रशासन को व्यवक्स्थत
ककया, और राज्य का पवस्तार और समेककत ककया। उन्होंने राजिानी को पार्न से नए शहर
अहमदाबाद में स्थानाींतररत कर ददया, क्जसकी नीींव उन्होंने 1413 में रखी थी। अहमदाबाद में जामा
मक्स्जद और तीन दरवाजा उनके समय की वास्तक
ु ला की शैली के बेहतरीन उदाहरण हैं।

Target PCS Lucknow Page 70


General Studies Paper#1-Sectional Paper#1

116. असभकथन (A): मोहम्मद ब्रबन ति


ु लक के शासनकाल के दौरान इब्न बतत
ू ा ने भारत का दौरा
ककया।

कारण (R): कफरोज शाह ति


ु लक पहले सल्
ु तान थे क्जन्होंने योग्यता के आिार पर लोिों को उच्च पद
ददए।

उपरोतत के सन्दभट में तनम्नसलखखत में से कौन सा सही है

A. A और R दोनों सत्य हैं और R, A की सही व्याख्या करता है ।

B. A और R दोनों सत्य हैं लेककन R, A की सही व्याख्या नहीीं है ।

C. A सच है लेककन R िलत है ।

D. A िलत है लेककन R सत्य है

उत्तर. C

स्पष्र्ीकरण: मोहम्मद ब्रबन ति


ु लक के काल में मोरतको का यात्री इब्न बतत
ू ा भारत आया था।
उन्होंने "रे हला" नामक एक पस्
ु तक सलखी क्जसमें उन्होंने वणटन ककया कक भारत में सती प्रथा प्रचसलत
थी। मोहम्मद ब्रबन ति
ु लक पहले सल्
ु तान थे क्जन्होंने कुलीन मल
ू पर योग्यता की वकालत की।

117. 'सल्तनत काल के आगथटक जीवन' के बारे में तनम्नसलखखत कथनों पर पवचार करें :

1. िज
ु रात में कैम्बे सोने और चाींदी के काम के सलए प्रससद्ि थे।

2. बींिाल का सोनारिााँव रे शम और मलमल के सलए प्रससद्ि था।

3. चरखा के आने से वस्त्रों के उत्पादन में सि


ु ार हुआ।

ऊपर ददए िए कथनों में से कौन-सा/से सही है /हैं?

A. केवल 1

B. केवल 2 और 3

C. उपरोतत सभी

D. उपरोतत में से कोई नहीीं

Target PCS Lucknow Page 71


General Studies Paper#1-Sectional Paper#1

उत्तर. C

स्पष्र्ीकरण: ददल्ली के सल्


ु तानों, रईसों, राज्यपालों, व्यापाररयों और समाज के कुलीनों के पास अपार
सींपपत्त थी और वे भौततक जीवन के सभी सख
ु ों का आनींद लेते थे। इस अवगि के दौरान कई खूबसरू त
मक्स्जदों, स्मारकों, महलों, ककलों और मींददरों का तनमाटण ककया िया। सल्
ु तानों के अिीन अींतदे शीय
और पवदे शी व्यापार फला-फूला। जहााँ तक आन्तररक व्यापार का प्रचन है हमारे पास व्यापाररयों और
दक
ु ानदारों के पवसभन्न विट थे। मख्
ु य रूप से उत्तर के िज
ु राती, दक्षिण के चेट्र्ी, राजपत
ू ाना के बींजारे
मख्
ु य व्यापारी थे। मींडडयों में वस्तओ
ु ीं के बडे सौदे ककए जाते थे।

118. तनम्नसलखखत कथनों पर पवचार कीक्जएः

1. इतता एक िेत्रीय कायट था और इसके िारक को मक्ु तत कहा जाता था।

2. वह िेत्र क्जसका राजस्व सीिे सल्


ु तान के अपने खजाने के सलए एकत्र ककया जाता था, को खलीसा
नासमत ककया िया था।

3. अलाउद्दीन खखलजी के अिीन खलीसा का आकार काफी बढ िया था।

ऊपर ददए िए कथनों में से कौन-से सही हैं?

A. केवल 1

B. केवल 2 और 3

C. केवल 1 और 3

D. 1, 2 और 3

उत्तर. D

स्पष्र्ीकरण: इतता की शरु


ु आत इल्तत
ु समश ने की थी। इसके िारक को मक्ु तत या वाली कहा जाता
था। मत
ु ती सल्
ु तान की ओर से भसू म कर और अन्य करों को उगचत तरीके से वसल
ू करने का हकदार
था, उसका व्यक्तत, मदहलाओीं और बच्चों, भसू म या काचतकारों की अन्य सींपपत्त पर कोई और दावा
नहीीं था। मत
ु ती के सल्
ु तान के प्रतत कुछ दातयत्व थे, मख्
ु य दातयत्व सैतनकों का रखरखाव और
सल्
ु तान के आह्वान पर उन्हें प्रस्तत
ु करना था। इतता एक हस्ताींतरणीय प्रभार था और इतता के
हस्ताींतरण अतसर होते थे।

Target PCS Lucknow Page 72


General Studies Paper#1-Sectional Paper#1

119. बलबन के बारे में तनम्नसलखखत में से कौन से कथन सही हैं?

1. बलबन ने लिातार राजशाही की प्रततष्ठा और शक्तत को बढाने की माींि की।

2. इस बात पर जोर दे ने के सलए कक रईस उसके बराबर नहीीं थे, उसने ससजादा और पाइबोस के
समारोहों पर जोर ददया।

3. उन्होंने महत्वपण
ू ट सरकारी पदों के सलए ऐसे ककसी भी व्यक्तत पर पवचार करने से इींकार कर ददया
जो एक कुलीन पररवार से नहीीं था।

4. उसने चहलिानी (तक


ु ी रईसों) की शक्तत को तोडा।

नीचे ददए िए कूर् का प्रयोि कर सही उत्तर चुतनए:

A. केवल 1, 2 और 4 4

B. केवल 2, 3 और 4

C. केवल 1 और 3

D. 1, 2, 3 और 4

उत्तर. D

स्पष्र्ीकरण: बलबन ने लिातार राजशाही की प्रततष्ठा और शक्तत को बढाने की कोसशश की, तयोंकक
उसे पवचवास था कक उसके सामने आने वाले आींतररक और बाहरी खतरों का सामना करने का यही
एकमात्र तरीका है । उन्होंने ससजादा और पाइबोस (साष्र्ाींि प्रणाम और सम्रार् के पैर चुींबन) के चलन
पर जोर ददया। बलबन ने सैन्य पवभाि (दीवान-ए-अजट) का पन
ु िटठन ककया। उन्होंने महत्वपण
ू ट
सरकारी पदों के सलए ककसी भी व्यक्तत पर पवचार करने से इींकार कर ददया जो एक कुलीन पररवार
से सींबगीं ित नहीीं था। इसका वस्तत
ु ः अथट भारतीय मस
ु लमानों को सत्ता और अगिकार के सभी पदों से
बदहष्कृत करना था। बलबन अींतत: चहलिानी, यानी तक
ु ी रईसों की शक्तत को तोडने और राजशाही
की शक्तत और प्रततष्ठा को बढाने के सलए दृढ था।

120. ससकींदर लोदी के सींबि


ीं में तनम्नसलखखत में से कौन सा कथन सही है ?

1. उनके शासनकाल में वास्को डी िामा भारत में उतरा।

2. उन्होंने िज-ए-ससकींदरी नामक याडट की एक नई माप स्थापपत की, जो मि


ु ल काल तक जारी रही।

Target PCS Lucknow Page 73


General Studies Paper#1-Sectional Paper#1

3. उनके प्रयासों के कारण, कई सींस्कृत कायों का फारसी में अनव


ु ाद ककया िया।

नीचे ददए िए कूर् का प्रयोि कर सही उत्तर चुतनए:

A. केवल 1 और 3

B. केवल 1 और 2

C. केवल 2 और 3

D. 1, 2 और 3

उत्तर. D

स्पष्र्ीकरण: सबसे महत्वपण


ू ट लोदी सल्
ु तान ससकींदर लोदी (1489-1517) था। वह िज
ु रात के महमद

बेिडा और मेवाड के राणा साींिा के समकालीन थे। सल्
ु तान ने कृपष में िहरी रुगच ली। उन्होंने अनाज
पर चींि
ु ी शल्
ु क को समाप्त कर ददया और याडट का एक नया माप स्थापपत ककया, क्जसे िाजी-
ससकींदरी कहा जाता है , जो मि
ु ल काल तक जारी रहा। ससकींदर लोदी ने पवद्वानों, दाशटतनकों और
सादहत्यकारों को शानदार अनद
ु ान ददया, क्जससे अरब और ईरान सदहत सभी दे शों के सस
ु स्
ीं कृत लोि
उसके दरबार में आते थे। सल्
ु तान के प्रयासों के कारण, कई सींस्कृत कायों का फारसी में अनव
ु ाद
ककया िया। ससकींदर लोदी, क्जनके शासनकाल में वास्को डी िामा भारत में उतरे , लोदी शासकों में
सबसे शक्ततशाली थे। उसने ब्रबहार और बींिाल पर पवजय प्राप्त की, ग्वासलयर को अपने अिीन कर
सलया और 1504 में आिरा शहर की स्थापना की।

वर्जयनगर िाम्राज्य और बहर्नी िाम्राज्य

121. बहमनी साम्राज्य के दौरान, अफाककस, क्जन्हें ग़रीब भी कहा जाता था, थे

A. राजाओीं द्वारा जारी ककए िए सोने के ससतकों की ढलाई में शासमल।

B. नवािींतक
ु जो पवदे शी थे।

C. दे हात में पवशाल भ-ू स्वामी।

D. वे जो रे ड यतू नयनों या गिल्डों का नेतत्ृ व करते थे और बैंकरों और पवत्तपोषकों के रूप में कायट
करते थे।

Target PCS Lucknow Page 74


General Studies Paper#1-Sectional Paper#1

उत्तर. B

स्पष्र्ीकरण: बहमनी साम्राज्य के सामने सबसे कदठन समस्याओीं में से एक रईसों के बीच सींघषट था।
रईसों को लींबे समय से स्थापपत दतकतनयों और नवािींतक
ु ों में पवभाक्जत ककया िया था जो पवदे शी थे
(अफाककयों, क्जन्हें ग़रीब भी कहा जाता है )। इराक और ईरान के नवािींतक
ु ों को ग़रीबदु दयार या
अफाककस कहा जाता था क्जसका अथट महानिरीय था और हबासशयों के साथ उत्तर के मल
ू प्रवाससयों
को दतकन या दक्षिणी कहा जाता था। इन दो समह
ू ों ने कुलीन विट का िठन ककया।

122. कुछ इततहासकार कृष्णदे व राय को पवजयनिर के सभी शासकों में सबसे महान मानते हैं।
कृष्ण दे व राय तनम्नसलखखत में से ककस राजवींश से सींबगीं ित थे?

A. चालत
ु य वींश

B. चोल वींश

C. सींिम वींश

D. तल
ु व
ु ा राजवींश

उत्तर. D

स्पष्र्ीकरण: कृष्ण दे व द्वारा एक नए राजवींश (तल


ु व
ु वींश कहा जाता है ) की स्थापना की िई थी।
कृष्णदे व राय (1509–30) इस राजवींश के सबसे महान व्यक्तत थे। कुछ इततहासकार उन्हें पवजयनिर
के सभी शासकों में सबसे महान मानते हैं। कृष्ण दे व को न केवल आींतररक कानन
ू और व्यवस्था को
कफर से स्थापपत करना था, उसे पवजयनिर के परु ाने प्रततद्वींद्पवयों से भी तनपर्ना था।

123. असभकथन (A): कफरोज शाह बहमनी ने खिोल पवज्ञान की खोज को प्रोत्सादहत ककया और
दौलताबाद के पास एक वेिशाला का तनमाटण ककया।

कारण (R): वह सभी िमों के ससद्िाींतों का सम्मान करता था।

उपरोतत के सन्दभट में तनम्नसलखखत में से कौन सा सही है

A. A और R दोनों सत्य हैं और R, A की सही व्याख्या करता है ।

B. A और R दोनों सत्य हैं लेककन R, A की सही व्याख्या नहीीं है ।

Target PCS Lucknow Page 75


General Studies Paper#1-Sectional Paper#1

C. A सच है लेककन R िलत है ।

D. A िलत है लेककन R सत्य है

उत्तर. B

स्पष्र्ीकरण: कफरोज शाह बहमनी िासमटक पवज्ञान, यानी कुरान, न्यायशास्त्र, आदद पर दर्प्पखणयों से
अच्छी तरह से पररगचत थे, और पवशेष रूप से वनस्पतत पवज्ञान, ज्यासमतत, तकट, आदद जैसे प्राकृततक
पवज्ञानों के शौकीन थे। वह एक अच्छे सल
ु ेखक और एक कपव थे, और अतसर अततशयोक्ततपण
ू ट छीं दों
की रचना करते थे। फररचता (यात्री) के अनस
ु ार, वह न केवल फारसी, अरबी और तक
ु ी में , बक्ल्क
तेलि
ु ,ु कन्नड और मराठी में भी पारीं ित था। कफरोज शाह बहमनी द्वारा उठाया िया सबसे
उल्लेखनीय कदम प्रशासन में दहींदओ
ु ीं को बडे पैमाने पर शासमल करना था। उसने अपने राज्य के
प्रमख
ु बींदरिाहों, चौल और दाभोल पर बहुत ध्यान ददया, जो फारस की खाडी और लाल सािर से
व्यापाररक जहाजों को आकपषटत करते थे, और दतु नया के सभी दहस्सों से पवलाससता के सामान लाते
थे।

124. तनम्नसलखखत में से ककस पवदे शी यात्री ने पवजयनिर साम्राज्य का दौरा ककया?

1. डोसमींिो पेस

2. डुआर्े बारबोसा

3. फनाटओ नतु नजो

नीचे ददए िए कूर् का प्रयोि कर सही उत्तर चुतनए।

A. केवल 1 और 2

B. केवल 2 और 3

C. 1 केवल

D. 1, 2 और 3

उत्तर. D

Target PCS Lucknow Page 76


General Studies Paper#1-Sectional Paper#1

स्पष्र्ीकरण: कृष्णदे व ने तेलि


ु ,ु कन्नड और तसमल कपवयों को समान रूप से अपना सींरिण ददया।
ड्यरू े र् बारबोसा, डोसमींिो पेस और फनाटओ नतु नज जैसे पवदे शी यात्री उसके कुशल प्रशासन और उसके
अिीन साम्राज्य की समद्
ृ गि की बात करते हैं।

125. पवजयनिर साम्राज्य के पतन के कारणों के सींदभट में , तनम्नसलखखत कथनों पर पवचार कीक्जएः

1. सेना प्रमख
ु ों में पविोह था।

2. कृष्णदे व राय की मत्ृ यु के बाद, उनके ररचतेदारों के बीच उत्तरागिकार के सलए सींघषट चल रहा था,
तयोंकक उनके सभी बेर्े नाबासलि थे।

3. मि
ु लों द्वारा आिमण।

ऊपर ददए िए कथनों में से कौन-से सही हैं?

A. केवल 1 और 2

B. केवल 2 और 3

C. केवल 1 और 3

D. 1, 2 और 3

उत्तर. A

स्पष्र्ीकरण: कृष्णदे व की मत्ृ यु (1530) के बाद, उनके सींबगीं ियों के बीच उत्तरागिकार के सलए सींघषट
चल रहा था, तयोंकक उनके सभी पत्र
ु नाबासलि थे। उसके उत्तरागिकारी पविोही नायक या सेना प्रमख
ु ों
से परे शान थे। 1542 तक, केंि पर तनयींत्रण एक अन्य शासक वींश में स्थानाींतररत हो िया था, जो
कक अरापवद ु का था, जो 17वीीं शताब्दी के अींत तक सत्ता में रहा। इस अवगि के दौरान, वास्तव में
पहले की तरह, पवजयनिर के शासकों के साथ-साथ दतकन सल्तनत के शासकों की सैन्य
महत्वाकाींिाओीं के पररणामस्वरूप सींरेखण में बदलाव आया। आखखरकार इसने पवजयनिर के खखलाफ
सल्तनत के िठबींिन का नेतत्ृ व ककया। उस ततगथ तक मि
ु लों ने दक्षिण में अपने साम्राज्य का
पवस्तार नहीीं ककया था।

र्ुगल िाम्राज्य

Target PCS Lucknow Page 77


General Studies Paper#1-Sectional Paper#1

126. तनम्नसलखखत में से ककस मि


ु ल शासक ने मराठों को चौथ नहीीं बक्ल्क दतकन के सरदे श मख
ु ी
को इकट्ठा करने का अगिकार दे कर ससख और मराठा के बीच आत्मीयता पैदा करने की कोसशश की?

A. जहााँदर शाह

B. मअ
ु ज्जम

C. अकबर

D. फारुखससयरी

उत्तर. B

स्पष्र्ीकरण: बहादरु शाह, क्जन्हें मह


ु म्मद मअ
ु ज्जम और शाह आलम के नाम से भी जाना जाता है ,
भारत के आठवें मि
ु ल सम्रार् थे, क्जन्होंने 1707 से 1712 में अपनी मत्ृ यु तक शासन ककया। अपनी
यव
ु ावस्था में , उन्होंने अपने पपता औरीं िजेब, छठे मि
ु ल सम्रार् को उखाड फेंकने की साक्जश रची।
मअ
ु ज्जम ने ससख और मराठा के बीच आत्मीयता पैदा करने की कोसशश की। उसने मराठों को
दतकन के सरदे श मख
ु ी को इकट्ठा करने का अगिकार ददया लेककन चौथ का नहीीं।

127. तनम्नसलखखत मि
ु ल बादशाहों में से कौन सैय्यद बींिुओीं के पतन का कारण बना?

A. बहादरु शाह-I

B. रफी-उद-दौलाह

C. शाहजहााँ-II

D. मह
ु म्मद शाह

उत्तर. D

स्पष्र्ीकरण: सैय्यद ब्रदसट अब्दल्


ु ला खान (वजीर) और हुसैन अली खान (मीर बख्शी) मि
ु ल सेना में
सेनापतत थे। सम्रार् औरीं िजेब की मत्ृ यु के बाद वे बहुत प्रभावशाली हो िए। जब मग़
ु ल बादशाह
मह
ु म्मद शाह िद्दी पर बैठा तो उसने भी परू ा तनयींत्रण अपने हाथ में ले सलया और मारे िए लोिों
को पकड सलया। उन्हें "ककीं ि मेकर" के रूप में भी जाना जाता है । मह
ु म्मद शाह (1719-1748) 1720
तक सैय्यद ब्रदसट की मदद से सत्रह साल की उम्र में ससींहासन पर चढा। मह
ु म्मद शाह ने अपने
शासन को वापस लेने के सलए तनजाम-उल-मल्
ु क आसफ जाह की मदद से भाइयों को मारने की

Target PCS Lucknow Page 78


General Studies Paper#1-Sectional Paper#1

व्यवस्था की। 1720 में फतेहपरु सीकरी में सैयद हुसैन अली खान की हत्या कर दी िई थी, और
सैयद हसन अली खान बरहा को 1722 में घातक रूप से जहर ददया िया था।

128. हल्दीघार्ी के यद्


ु ि में मि
ु ल सेना की कमान ककसके द्वारा ली िई थी?

A. आसफ खान

B. अींबरी के राजा मान ससींह

C. काजी खान

D. र्ोडर मल

उत्तर. B

स्पष्र्ीकरण: हल्दीघार्ी की लडाई 18 जून 1576 को मेवाड के राणा, महाराणा प्रताप का समथटन
करने वाले घड
ु सवारों और िनि
ु ाटररयों के बीच लडी िई लडाई थी और मि
ु ल सम्रार् अकबर की सेनाएीं
क्जनका नेतत्ृ व अींबर के मान ससींह प्रथम ने ककया था। महाराणा प्रताप ने वीरता से यद्
ु ि लडा,
लेककन मि
ु ल सेना से हार िए।

129. शेर शाह सरू ी ने ककस मि


ु ल सम्रार् को पराक्जत ककया था?

A. बाबर

B. हुमायूाँ

C. अकबर

D. जहाींिीर

उत्तर. B

स्पष्र्ीकरण: मि
ु ल वींश के सींस्थापक बाबर के लोददयों को हराने के बाद, शेर शाह सरू ी ने ब्रबहार और
बींिाल के अफिान राज्यों पर तनयींत्रण प्राप्त कर सलया और चौसा (1539) और कन्नौज (1540) में
मि
ु ल सम्रार् हुमायूाँ को हराया। शेर शाह ने परू े उत्तर भारत पर पाींच साल तक शासन ककया, मालवा
पर कब्जा कर सलया और राजपत
ू ों को हराया। चौसा की लडाई मि
ु ल सम्रार् हुमायूीं और अफिान शेर
शाह सरू ी के बीच एक उल्लेखनीय यद्
ु ि था। यह 26 जून 1539 को भारत के आितु नक ब्रबहार में

Target PCS Lucknow Page 79


General Studies Paper#1-Sectional Paper#1

बतसर से 10 मील दक्षिण-पक्चचम में चौसा में लडा िया था। हुमायूाँ अपनी जान बचाने के सलए यद्
ु ि
के मैदान से भाि तनकला। शेर शाह पवजयी हुआ और उसने खद
ु को फरीद अल-दीन शरू शाह का
ताज पहनाया।

130. मि
ु ल काल में कृपष भसू म के रक्जस्र्रों का रखरखाव ककसके द्वारा ककया जाता था?

A. कोतवाली

B. कानन
ू िो

C. अमीन

D. िोररक

उत्तर. B

स्पष्र्ीकरण: पण
ू ट शक्तत का आनींद लेते हुए, मि
ु ल साम्राज्य का सम्रार् हमेशा केंिीय प्रशासतनक
अगिकारी था। पवसभन्न मामलों से जड
ु े लेनदे न के सच
ु ारू सींचालन के सलए पवसभन्न सरकारी पवभािों
में कई अगिकाररयों को तनयत
ु त ककया िया था। कानन
ू िो परिने के पर्वाररयों का मखु खया था और
फसलों, राजस्व माींिों, वास्तपवक भि
ु तान, बकाया आदद का ररकॉडट रखता था।

131. बल
ु द
ीं दरवाजा, अकबर द्वारा फतेहपरु सीकरी में अपनी ककस पर जीत के उपलक्ष्य में बनवाया
िया था?

A. बरारी

B. मेवाड

C. िज
ु रात

D. बींिाल

उत्तर. C

स्पष्र्ीकरण: बल
ु द
ीं दरवाजा, या "जीत का द्वार", मि
ु ल सम्रार् अकबर द्वारा िज
ु रात पर अपनी जीत
के उपलक्ष्य में 1575 ईस्वी में बनाया िया था। यह फतेहपरु सीकरी में जामा मक्स्जद का मख्
ु य

Target PCS Lucknow Page 80


General Studies Paper#1-Sectional Paper#1

प्रवेश द्वार है , जो भारत के आिरा से 43 ककमी दरू है । बल


ु द
ीं दरवाजा दतु नया का सबसे ऊींचा प्रवेश
द्वार है और मि
ु ल वास्तक
ु ला का एक उदाहरण है ।

132. जहाींिीर के शासनकाल में ककस ससख िरु


ु की शहादत हुई थी?

A. िरु
ु नानक ससींह जी

B. िरु
ु िोब्रबींद ससींह जी

C. िरु
ु अजटन ससींह जी

D. िरु
ु तेि बहादरु ससींह जी

उत्तर. C

स्पष्र्ीकरण: 16 जून, 1606 को सम्रार् जहाींिीर के नेतत्ृ व वाली मि


ु ल सरकार द्वारा पाींच ददनों तक
प्रताडडत ककए जाने के बाद िरु
ु अजटन की मत्ृ यु हो िई थी। ससख हर साल 16 जून को ससख िरु

अजटन की शहादत का जचन मनाते हैं। उनकी शहादत को िरु
ु अजटन के शहीदी ददवस के रूप में याद
ककया जाता है ।

133. कलानौर को -------- के स्थान के रूप में जाना जाता है

A. अकबर को बादशाह घोपषत करना

B. अकबर का जन्म स्थान

C. हुमायूाँ का पववाह समारोह

D. उपरोतत में से कोई नहीीं

उत्तर. A

स्पष्र्ीकरण: कलानौर को बादशाह के रूप में अकबर की घोषणा के स्थान के रूप में जाना जाता है ।
पींजाब के कलानौर में , 13 वषीय अकबर को बैरम खान द्वारा एक नवतनसमटत मींच पर पवराजमान
ककया िया था। उन्हें शहीं शाह ("राजाओीं के राजा" के सलए फारसी शब्द) घोपषत ककया िया था। बैरम
खााँ ने उसकी ओर से तब तक शासन ककया जब तक वह बडा नहीीं हुआ।

Target PCS Lucknow Page 81


General Studies Paper#1-Sectional Paper#1

134. तनम्नसलखखत में से कौन मि


ु ल भारत में फसलों के सींबि
ीं में क्जन्स-ए-कासमल शब्द का सही
अथट दशाटता है ?

A. िान की फसल

B. नकद फसल

C. मोर्े फसल

D. शष्ु क िेत्र में उिाई जाने वाली फसल

उत्तर. B

स्पष्र्ीकरण: इसका अथट है उत्तम फसलें। यह नकदी फसलों को सींदसभटत करता है । चूींकक वे दे श में
आय लाते थे, मि
ु ल शासकों ने ककसानों को नकदी फसलों के सलए प्रोत्सादहत ककया। सबसे महत्वपण
ू ट
क्जन्स-ए-कासमल कपास और िन्ना थे। मध्य भारत और दतकन के उच्चभसू म में कपास की खेती की
जाती थी। बींिाल िन्ने के सलए प्रससद्ि था। सरसों की तरह दलहन और ततलहन को भी नकदी
फसल माना जाता था। यह दशाटता है कक दे श में खाद्य फसलों और नकदी फसलों का समश्रण था।

135. दारा सशकोह अींतत: औरीं िजेब से उत्तरागिकार का यद्


ु ि ककस यद्
ु ि में हार िया-

A. िरमत

B. समि
ु ढ

C. दे वराय

D. खानआ

उत्तर. B

स्पष्र्ीकरण: समि
ु ढ की लडाई, जींि-ए-समि
ु ढ, ससतींबर 1657 में सम्रार् की िींभीर बीमारी से मत्ृ यु के
बाद मि
ु ल सम्रार् शाहजहाीं के पत्र
ु ों के बीच उत्तरागिकार के मि
ु ल यद्
ु ि के दौरान ससींहासन के सलए
सींघषट में एक तनणाटयक लडाई थी। उत्तरागिकार का यद्
ु ि औरीं िजेब की जीत के साथ समाप्त हुआ।

Target PCS Lucknow Page 82


General Studies Paper#1-Sectional Paper#1

136. अकबर के दरबार में प्रससद्ि सींिीतकार तानसेन का मल


ू नाम तनम्नसलखखत में से कौन सा
था?

A. महानींदा पाींडे

B. लाल कलावींती

C. बाज बहादरु

D. रामतनु पाींडे

उत्तर. D

स्पष्र्ीकरण: अकबर के दरबार में सबसे प्रससद्ि सींिीतकार तानसेन थे क्जनका मल


ू नाम रामतनु
पाींडे था। स्वामी हररदास तानसेन के महान िरु
ु थे क्जन्होंने उनकी सींिीत प्रततभा को पवकससत करने
में मदद की। तानसेन का नाम रुिवीना, समयाीं-की-तोडी (राि) और दरबारी काींडा (राि) की
खोज/नवाचार से जुडा है ।

137. ककस मि
ु ल शासक ने अपने शाींततपण
ू ट व्यक्ततत्व और िैयट के सलए इींसान-ए-कासमल (परफेतर्
मैन) की उपागि अक्जटत की?

A. शाहजहााँ

B. हुमायूाँ

C. जहाींिीर

D. अकबर

उत्तर. B

स्पष्र्ीकरण: मि
ु ल सम्रार् के शासक हुमायूीं ने इींसान-ए-कासमल (परफेतर् मैन) की उपागि प्राप्त की।
उनके शाींततपण
ू ट व्यक्ततत्व, िैयट और भाषण के िैर-उत्तेजक तरीकों ने उन्हें मि
ु लों के बीच 'इींसान-ए-
कासमल' ('परफेतर् मैन') की उपागि दी। वह बाबर का पत्र
ु और अकबर का पपता था।

138. मनसबदारों के वेतन को कहा जाता था

Target PCS Lucknow Page 83


General Studies Paper#1-Sectional Paper#1

A. मनसब

B. जातत

C. जािीर

D. जब्ती

उत्तर. C

स्पष्र्ीकरण: मनसबदारी प्रणाली मि


ु ल शासकों द्वारा एक मनसबदार की रैंक और वेतन तय करने
के सलए इस्तेमाल की जाने वाली एक ग्रेडडींि प्रणाली थी उन्हें मि
ु ल सम्रार् द्वारा तनयत
ु त और
बखाटस्त कर ददया जाता था। मनसबदारों को उनका वेतन जािीर नामक राजस्व कायट के रूप में प्राप्त
होता था। अगिकाींश मनसबदार वास्तव में अपनी जािीरों में तनवास या प्रशासन नहीीं करते थे। उनके
पास केवल अपने िेत्र के राजस्व का अगिकार था जो उनके नौकरों द्वारा उनके सलए एकत्र ककए
जाते थे।

139. सच
ू ी I को सच
ू ी II के साथ सम
ु ेसलत करें और नीचे ददए िए कूर् से सही उत्तर का चयन करें :

सच
ू ी I सच
ू ी II

(महत्वपण
ू ट पवभाि) (कायट)

a. दीवान-ए-वजरत 1. खुकफया पवभाि

b. दीवान-ए-रसलातमह
ु ताससब 2. सरकारी पत्रों का सींरिक

c. दीवान-ए-इींशा 3. पवदे श मामलों का पवभाि

d. दीवान-ए-बारीद 4. राजस्व और पवत्त पवभाि

कोड:

a b c d

A. 4 3 2 1

B. 3 2 4 1

C. 2 3 4 1

Target PCS Lucknow Page 84


General Studies Paper#1-Sectional Paper#1

D. 3 4 1 2

उत्तर. A

स्पष्र्ीकरण:

महत्वपण
ू ट पवभाि कायट

दीवान-ए-वजरात राजस्व और पवत्त पवभाि

दीवान-ए-अजट सैन्य पवभाि

दीवान-ए-रसलातमह
ु ताससब पवदे श मामलों का पवभाि

दीवान-ए-इींशा सरकारी कािजात के सींरिक

दीवान-ए-कुजा न्याय पवभाि

दीवान-ए-बरीद खकु फया पवभाि

दीवान-ए-समान शाही घराने का प्रभारी पवभाि

140. सच
ू ी I को सच
ू ी II से सम
ु ेसलत करें और नीचे ददए िए कूर् से सही उत्तर का चयन करें :

सच
ू ी I सच
ू ी II

(मि
ु ल सैतनक) (प्रकार)

a. वालशादहस 1. शाही अींिरिक

b. पपयादिन 2. पैदल सैतनक

c. बराक - अींदाज 3. सज्जनो - सैतनक

d. अहदीस 4. बींदक
ू िारी

कोड:

a b c d

A. 1 2 4 3

Target PCS Lucknow Page 85


General Studies Paper#1-Sectional Paper#1

B. 3 2 4 1

C. 2 3 4 1

D. 3 4 1 2

उत्तर. A

141. असभकथन (A): अकबर ने अपने शासन के दौरान सल


ु -ए-कुल की नीतत का पालन ककया।

कारण (R): यह सदहष्णुता का पवचार था क्जसने अपने िेत्र में पवसभन्न िमों के लोिों के बीच
भेदभाव नहीीं ककया। यह नैततकता की एक प्रणाली पर केंदित था जो सावटभौसमक रूप से लािू थी।

उपरोतत के सन्दभट में तनम्नसलखखत में से कौन सा सही है

A. A और R दोनों सत्य हैं और R, A की सही व्याख्या करता है ।

B. A और R दोनों सत्य हैं लेककन R, A की सही व्याख्या नहीीं है ।

C. A सच है लेककन R िलत है ।

D. A िलत है लेककन R सत्य है

उत्तर. B

स्पष्र्ीकरण: अकबर ने सल
ु ह-ए-कुल या "सावटभौसमक शाींतत" की नीतत का पालन ककया। यह
सदहष्णुता का पवचार था क्जसने अपने िेत्र में पवसभन्न िमों के लोिों के बीच भेदभाव नहीीं ककया।
इसके बजाय, इसने नैततकता की एक प्रणाली पर ध्यान केंदित ककया - ईमानदारी, न्याय, शाींतत - जो
सावटभौसमक रूप से लािू थी। अबल
ु फजल ने सल
ु ह-ए-कुल के इस पवचार के इदट-गिदट शासन की दृक्ष्र्
तैयार करने में अकबर की मदद की। इसका उपयोि जहााँिीर और शाहजहााँ ने भी ककया था।

142. असभकथन (A): पानीपत की पहली लडाई (1526) के बाद बाबर ने मि


ु ल साम्राज्य की नीींव
रखी।

कारण (R): भारत में बाबर का आिमन महत्वपण


ू ट था तयोंकक भौिोसलक दृक्ष्र् से, काबल
ु और कींिार
व्यापार मािट में क्स्थत थे, इससलए, इन दो िेत्रों के तनयींत्रण ने भारत के पवदे शी व्यापार को मजबत

ककया।

Target PCS Lucknow Page 86


General Studies Paper#1-Sectional Paper#1

उपरोतत के सन्दभट में तनम्नसलखखत में से कौन सा सही है

A. A और R दोनों सत्य हैं और R, A की सही व्याख्या करता है ।

B. A और R दोनों सत्य हैं लेककन R, A की सही व्याख्या नहीीं है ।

C. A सच है लेककन R िलत है ।

D. A िलत है लेककन R सत्य है ।

उत्तर. B

स्पष्र्ीकरण: काबल
ु और कींिार ने हमेशा भारत में आिमण के सलए मींचन स्थल के रूप में काम
ककया था। बाबर के आिमन ने काबल
ु और कींिार को उत्तर भारत के साम्राज्य का असभन्न अींि बना
ददया। बाबर और उसके उत्तरागिकाररयों ने बाहरी आिमण से भारत की सरु िा को मजबत
ू ककया, जो
पपछले 200 वषों से लिातार जारी था।

143. असभकथन (A): अलाउद्दीन खखलजी के शासनकाल के दौरान, शाहना व्यापाररयों का एक


रक्जस्र्र रखता था, और दक
ु ानदारों और कीमतों को सख्ती से तनयींब्रत्रत करता था।

कारण (R): इततहासकार बरनी ने सोचा कक अलाउद्दीन के बाजारों पर तनयींत्रण का एक प्रमख



उद्दे चय दहींदओ
ु ीं को दीं डडत करने की उसकी इच्छा थी।

उपरोतत के सन्दभट में तनम्नसलखखत में से कौन सा सही है

A. A और R दोनों सत्य हैं और R, A की सही व्याख्या करता है ।

B. A और R दोनों सत्य हैं लेककन R, A की सही व्याख्या नहीीं है ।

C. A सच है लेककन R िलत है ।

D. A िलत है लेककन R सत्य है

उत्तर. B

स्पष्र्ीकरण: समकालीन लोिों के सलए, अलाउद्दीन के बाजारों को तनयींब्रत्रत करने के उपाय दतु नया के
महान आचचयों में से एक थे। गचत्तौड असभयान से लौर्ने के बाद, अलाउद्दीन ने अनाज, चीनी और
खाना पकाने के तेल से लेकर सई
ु तक, और महीं िे आयाततत कपडे से लेकर घोडों, मवेसशयों और दास
लडकों लडककयााँ तक सभी वस्तओ
ु ीं की लाित तय करने की माींि की। इस उद्दे चय के सलए, उन्होंने

Target PCS Lucknow Page 87


General Studies Paper#1-Sectional Paper#1

ददल्ली में तीन बाजार स्थापपत ककए, एक खाद्यान्न के सलए, दस


ू रा महीं िे कपडे के सलए और तीसरा
घोडों, दासों और मवेसशयों के सलए। प्रत्येक बाजार शाहना नामक एक उच्च अगिकारी के तनयींत्रण में
था, जो व्यापाररयों का एक रक्जस्र्र रखता था, और दक
ु ानदारों और कीमतों को सख्ती से तनयींब्रत्रत
करता था। इततहासकार बरनी ने सोचा कक अलाउद्दीन के बाजारों पर तनयींत्रण का एक प्रमख
ु उद्दे चय
दहींदओ
ु ीं को दीं डडत करने की उनकी इच्छा थी, तयोंकक अगिकाींश व्यापारी दहींद ू थे और वे ही थे क्जन्होंने
खाद्यान्न और अन्य सामानों में मन
ु ाफाखोरी का सहारा सलया था। हालााँकक, पक्चचम और मध्य
एसशया में अगिकाींश भसू म व्यापार खुरसातनयों के हाथों में था, जो मक्ु स्लम थे, साथ ही मल्
ु तातनयों के
भी, क्जनमें से कई मस
ु लमान थे। इससलए, अलाउद्दीन के उपायों ने इन विों को भी प्रभापवत ककया,
एक तथ्य क्जसका बरनी उल्लेख नहीीं करता है ।

144. असभकथन (A): मि


ु ल पें दर्ींि में मैचलॉक को दशाटया िया है ।

कारण (R): अबल


ु फजल अकबर के शस्त्रािार में लोहे की तोपों और हथकडी के बैरल बनाने की
तकनीक का वणटन करता है ।

उपरोतत के सन्दभट में तनम्नसलखखत में से कौन सा सही है

A. A और R दोनों सत्य हैं और R, A की सही व्याख्या करता है ।

B. A और R दोनों सत्य हैं लेककन R, A की सही व्याख्या नहीीं है ।

C. A सच है लेककन R िलत है ।

D. A िलत है लेककन R सत्य है

उत्तर. B

स्पष्र्ीकरण: भारत के कुछ िेत्रों जैसे िज


ु रात, मालवा और दतकन में 15 वीीं शताब्दी के उत्तरािट के
दौरान पहली बार तछर्पर्
ु रूप से आग्नेयास्त्रों का उपयोि ककया िया था। लेककन दक्षिण भारत में
1498 ईसवी सन ् में पत
ु ि
ट ाली काल की एजेंसी के माध्यम से और उत्तर में 1526 ई. में बाबर द्वारा
तनयसमत रूप से आग्नेयास्त्रों का पवकास ककया िया। बाबर ने राजपत
ू ों और अफिानों के खखलाफ
लडाई में बींदक
ू ों और तोपों का इस्तेमाल ककया। ये बींदक
ू ें वास्तव में मैचलॉक थीीं। अबल
ु फजल
अकबर के शस्त्रािार में ब्रबना मैचलॉक के तनमाटण का दावा करता है । 16वीीं शताब्दी की पें दर्ींि, पवशेष
रूप से मि
ु ल सम्रार् अकबर के समय के गचत्रों में , कुछ सैतनकों को मैचलॉक का उपयोि करते हुए
गचब्रत्रत ककया िया है ।

Target PCS Lucknow Page 88


General Studies Paper#1-Sectional Paper#1

145. असभकथन (A): शाहजहााँ की कैद के ददनों में उसकी पसींदीदा बेर्ी जहााँआरा ने प्यार से उसकी
दे खभाल की थी।

कारण (R): औरीं िजेब के शासन के दौरान उनके फरमानों का सींग्रह 'जवाब्रबत-ए-आलमगिरी' नामक
कृतत में एकत्र ककया िया था।

उपरोतत के सन्दभट में तनम्नसलखखत में से कौन सा सही है

A. A और R दोनों सत्य हैं और R, A की सही व्याख्या करता है ।

B. A और R दोनों सत्य हैं लेककन R, A की सही व्याख्या नहीीं है ।

C. A सच है लेककन R िलत है ।

D. A िलत है लेककन R सत्य है

उत्तर. B

स्पष्र्ीकरण: जहााँआरा बेिम ददल्ली के प्रससद्ि चाींदनी चौक की लेखखका, कवतयत्री, गचत्रकार और
वास्तक
ु ार थीीं। सम्रार् शाहजहााँ और उनकी पसींदीदा पत्नी, मम
ु ताज महल की सबसे बडी सींतान,
जहााँआरा का जन्म 1614 में अजमेर में हुआ था। औरीं िजेब ने अींततः दारा सशकोह को मार डाला
और आिरा ककले के मथ
ु म्मन बज
ु ट (जैस्मीन र्ॉवर) में एक बीमार शाहजहााँ को नजरबींद कर ददया।
अपने पपता के प्रतत वफादार, जहााँआरा ने अपने आकषटक व्यापार और पवलाससतापण
ू ट जीवन शैली को
को छोड ददया। अपने तनवाटसन में शाहजहााँ के साथ एक तनरीं तर उपक्स्थतत, उसने आठ साल तक
उसकी दे खभाल की, जब तक कक उसने 1666 में अींततम साींस नहीीं ली। औरीं िजेब के फरमानों का
एक सींग्रह जवाब्रबत-ए-आलमगिरी के नाम से जाना जाता था। एक रूदढवादी मस
ु लमान होने के साथ-
साथ औरीं िजेब एक शासक भी था।

146. अकबर की िासमटक नीतत के बारे में तनम्नसलखखत कथनों पर पवचार करें :

1. महजर ने उलेमाओीं की शक्तत पर अींकुश लिाने के सलए उनके द्वारा प्रख्यापपत ककया था।

2. उन्होंने महादवी और सशयाओीं के प्रतत दमनकारी नीततयाीं अपनाईं।

3. इबादत खाना सभी प्रमख


ु िमों की भािीदारी के साथ िासमटक चचाटओीं के सलए खुला था।

Target PCS Lucknow Page 89


General Studies Paper#1-Sectional Paper#1

ऊपर ददए िए कथनों में से कौन-से सही हैं?

A. केवल 1 और 2

बी केवल 2 और 3

C. केवल 1 और 3

D. 1, 2 और 3

उत्तर. D

स्पष्र्ीकरण: इबादत खाना (1575) की स्थापना इस्लासमक िमटशास्त्र के पवसभन्न पहलओ


ु ीं पर स्वतींत्र
चचाट करने के उद्दे चय से की िई थी। लेककन, क्जस तरह से मक्ु स्लम न्यायपवद न्यायशास्त्र के सवालों
पर झिडते थे, उसी तरह सम्रार् का मोहभींि हो िया। शरु
ु आत में, केवल सक्ु न्नयों को चचाट में भाि
लेने की अनम
ु तत थी। 1578 से, सम्रार् ने सकू फयों, सशयाओीं, ब्राह्मणों, जैतनयों, ईसाइयों, यहूददयों,
पारससयों आदद के सलए इबादत खाना के द्वार खोल ददए। अकबर ने सभी िासमटक मामलों को अपने
हाथों में लेने के सलए महजरनामा (या अचक
ू ता फरमान) जारी ककया। इसने उन्हें िासमटक मामलों में
सवोच्च बना ददया। उसने उलेमाओीं के प्रभत्ु व पर अींकुश लिाने के सलए महजरनामा जारी ककया। यह
फैजी द्वारा 1579 ई. में सलखा िया था। उन्होंने सशयाओीं और महादपवयों के खखलाफ उत्पीडन की
नीतत का पालन ककया।

147. सफ
ू ी आींदोलन के बारे में तनम्नसलखखत कथनों पर पवचार करें :

1. अल हुजपवरी को भारत में सफ


ू ी आींदोलन का सींस्थापक माना जाता है ।

2. क्जयारत का तात्पयट सफ
ू ी सींतों की कब्रों की तीथटयात्रा से है ।

3. वहदत-उल-वज
ु द
ु सफ
ू ीवाद का एक मौसलक ससद्िाींत है , जो अक्स्तत्व की एकता में पवचवास करता
है ।

ऊपर ददए िए कथनों में से कौन-से सही हैं?

A. केवल 1 और 2

बी केवल 2 और 3

C. केवल 1 और 3

Target PCS Lucknow Page 90


General Studies Paper#1-Sectional Paper#1

D. 1, 2 और 3

उत्तर. D

स्पष्र्ीकरण: भारत में सफ


ू ी आींदोलन 11वीीं शताब्दी ईस्वी में शरू
ु हुआ था, अल हुजपवरी, क्जन्होंने
खुद को उत्तर भारत में स्थापपत ककया था, को लाहौर में दफनाया िया था और उपमहाद्वीप में सबसे
परु ाने सफ
ू ी के रूप में माना जाता था। क्जयारत सफ
ू ी सींतों की कब्रों पर तीथटयात्रा के रूप में जाने की
सफ
ू ी प्रथा को सींदसभटत करता है । वहदत अल-वज
ु ुद का अथट है अक्स्तत्व की एकता, यह इब्न अल-
अरबी के स्कूल द्वारा तैयार ककया िया ससद्िाींत है , जो यह बताता है कक ईचवर और उसकी रचना
एक है , तयोंकक जो कुछ भी बनाया िया है वह ईचवर के ज्ञान में पहले से मौजूद है और ईचवर के
साथ रहस्यमय समलन को सींभव बनाते हुए वापस आ जाएिा।

148. तौहीद-ए-इलाही, एक नया सफ


ू ी प्रकार का आदे श, अकबर द्वारा 1582 में शरू
ु ककया िया था।
तौहीद-ए-इलाही के सींबि
ीं में तनम्नसलखखत में से कौन सा कथन सही है / हैं?

1. यह िासमटक शास्त्रों की सवोच्चता में पवचवास करता था।

2. केवल अकबर द्वारा अनम


ु ोददत लोिों को ही सदस्य के रूप में शासमल होने की अनम
ु तत दी िई
थी।

3. सदस्यों को आदे श में शासमल होने के सलए एकमचु त मौदिक प्रोत्साहन का भि


ु तान ककया िया
था।

नीचे ददए िए कूर् का प्रयोि कर सही उत्तर चतु नए:

A. 2 केवल

B. केवल 1 और 3

C. 1 और 2 केवल

D. 1, 2 और 3

उत्तर. A

स्पष्र्ीकरण: तौहीद-ए-इलाही वास्तव में सफ


ू ीवादी प्रकार की एक शाखा थी। अकबर की खोज तौहीद-
ए-इल्ही (ददव्य एकेचवरवाद) या दीन-ए-इलाही में समाप्त हुई। जो लोि शासमल होने के इच्छुक थे,
और क्जन्हें सम्रार् ने मींजूरी दे दी थी, उन्हें सदस्य बनने की अनम
ु तत दी िई थी। रपववार को दीिा

Target PCS Lucknow Page 91


General Studies Paper#1-Sectional Paper#1

का ददन तनिाटररत ककया िया था। दीिाओीं को जहाीं तक सींभव हो माींस से परहे ज करना था, कम से
कम उनके जन्म के महीने में , उनके जन्मददन पर शानदार दावत और सभिा दे नी चादहए। दीिा के
अलावा कोई पपवत्र ग्रींथ या ग्रींथ नहीीं थे, कोई परु ोदहत विट नहीीं था, कोई पज
ू ा स्थल या अनष्ु ठान
और समारोह नहीीं थे। अकबर ने न तो बल प्रयोि ककया और न ही सशष्यों या मरु ीदों के नामाींकन के
सलए िन का प्रयोि ककया िया।

149. भारत की पवजय के सलए बाबर को प्रेररत करने वाले कारकों के बारे में तनम्नसलखखत कथनों पर
पवचार करें :

1. मध्य एसशया के अनगिनत आिमणकाररयों की तरह, बाबर अपनी शानदार सींपपत्त के लालच में
भारत की ओर आकपषटत हुआ।

2. बाबर काबल
ु पर एक उज़्बेक हमले से आशींककत था और भारत को शरण का अच्छा स्थान मानता
था।

3. काबल
ु की अल्प आय ने बाबर को भारत पर पवजय प्राप्त करने के सलए मजबरू ककया।

उपरोतत में से कौन सा/से कथन सही है /हैं?

A. केवल 1

B. केवल 1 और 2

C. केवल 2 और 3

D. उपरोतत सभी

उत्तर. D

स्पष्र्ीकरण: बाबर मि
ु ल साम्राज्य का सींस्थापक और भारतीय उपमहाद्वीप में मि
ु ल वींश का पहला
सम्रार् (1526-1530) था। वह िमशः अपने पपता और माता के माध्यम से तैमरू और चींिेज खान के
वींशज थे। उन्हें कफरदौस मकानी ('स्विट में तनवास') का मरणोपराींत नाम भी ददया िया था। भारत की
पवजय के सलए बाबर को प्रेररत करने वाले कारकों में से प्रमख
ु तनम्नसलखखत हैं :

1) जब बाबर समरकींद को जीतने में असफल रहा, तो उसने दहींदस्


ु तान की पवजय पर ध्यान केंदित
ककया।

2) भारत की वैभवशाली सींपदा उसके सलए बहुत आकषटण का स्रोत थी।

Target PCS Lucknow Page 92


General Studies Paper#1-Sectional Paper#1

3) उस समय भारत में राजनीततक अक्स्थरता थी।

150. मि
ु ल सम्रार् जहाींदार शाह के शासनकाल के बारे में तनम्नसलखखत कथनों पर पवचार करें :

1. नफरत करने वाले जक्जया को खत्म कर ददया िया।

2. अींबर के जय ससींह को समजाट राजा सवाई की उपागि दी िई और उन्हें िज


ु रात का राज्यपाल
तनयत
ु त ककया िया।

3. मारवाड के अजीत ससींह को महाराजा की उपागि से सम्मातनत ककया िया और उन्हें मालवा का
राज्यपाल तनयत
ु त ककया िया।

उपरोतत में से कौन सा/से कथन सही है /हैं?

A. केवल 1

B. केवल 1 और 2

C. केवल 2 और 3

D. उपरोतत सभी

उत्तर. A

स्पष्र्ीकरण: बहादरु शाह के कम सिम पत्र


ु ों में से एक, जहाींदार शाह ने उस समय के सबसे
शक्ततशाली कुलीन जुक्ल्फकार खान की सहायता और समथटन से ससींहासन जीता। राजकुमार को
प्रशासन में सबसे कम ददलचस्पी थी और यह जुक्ल्फकार के हाथों में चला िया। हालााँकक, उनकी
नीततयाीं चतरु और प्रिततशील थीीं। वह जानता था कक उसे राजपत
ू राजाओीं और मराठा सरदारों और
अन्य स्थानीय कुलों के साथ मैत्रीपण
ू ट सींबि
ीं स्थापपत करने होंिे। इससलए उन्होंने इस ददशा में
आवचयक कदम उठाए। मारवाड और अींबर की वे मााँिें क्जन पर बहादरु शाह ने सींतोषजनक ढीं ि से
ध्यान नहीीं ददया, उन्हें सींबोगित ककया िया। उसने जक्जया को समाप्त कर ददया। उसने मराठों को
दतकन का चौथ और सरदे शमख
ु ी भी प्रदान ककया। उनकी नीततयों में से एक इरजराह या राजस्व
खेती की शरू
ु आत थी, क्जससे लोि नफरत करते थे। जब रईसों को जुक्ल्फकार खान की क्स्थतत से
जलन हुई, तो उन्होंने उसके खखलाफ बादशाह के कानों में जहर भर ददया। इससे उनके बीच
िलतफहमी हो िई। अींत में , जहाींदार शाह को उसके भतीजे फारुख ससयार ने आिरा में पराक्जत
ककया। उसके शासनकाल में आमेर के जय ससींह को समजाट राजा सवाई की उपागि दी िई और उन्हें

Target PCS Lucknow Page 93


General Studies Paper#1-Sectional Paper#1

मालवा का राज्यपाल तनयत


ु त ककया िया। मारवाड के अजीत ससींह को महाराजा की उपागि दी िई
और उन्हें िज
ु रात का राज्यपाल तनयत
ु त ककया िया।

Target PCS Lucknow Page 94

You might also like